Gastro-intestinal System Flashcards
A 68-year-old man undergoes barium swallow for dysphagia. During the examination the
patient has an episode of coughing, and barium is noted to enter the larynx and proximal trachea.
What is the appropriate management? [B4 Q62]
a. no action needed
b. physiotherapy
c. prophylactic antibiotics
d. chest radiograph in 48 hours
e. admission to hospital for observation
Physiotherapy
Barium aspiration is a recognized complication of barium swallow and may occur particularly in patients with swallowing disorders or recent oesophageal surgery. It is usually clinically insignificant, but complications have been reported, especially with aspiration of larger amounts of barium, and include pneumonitis and granuloma formation. Physiotherapy is the only treatment recommended. Of the water-soluble contrast agents, Gastrografin (ionic and hyperosmolar) may cause pulmonary oedema if aspirated. Gastromiro (non-ionic and iso-osmolar) is safe to use if aspiration is a significant possibility.
A 32-year-old female patient attends for a barium swallow with a history of a sensation of food
sticking in her throat. The barium swallow reveals (1) uniform horizontally orientated folds in
the lower oesophagus. There is (2) a change in the texture of the mucosa 1 cm above the hiatus,
which is (3) sited 25 cm from the origin of the oesophagus. There is a (3) slight smooth
narrowing noted 2 cm above the hiatus, beyond which there is a (4) slight dilatation of the
oesophagus prior to it joining the stomach. Which of the following is an unusual finding? [B1
Q2]
A. The appearance of the oesophageal folds.
B. The change in mucosal appearance 1 cm above the hiatus.
C. The distance of the hiatus from the origin of the oesophagus.
D. The slight narrowing 2 cm above the hiatus.
E. The distal bulge just before the stomach.
The appearance of the oesophageal folds
The oesophageal folds are normally longitudinally orientated. Horizontally orientated folds are described as feline oesophagus.
The change in mucosal appearance is the normal Z line – the squamo-columnar junction. The narrowing described is the A line at the origin of the vestibule of the distal oesophagus. The position of the hiatus is normally stated as being 40 cm. This is the distance from the teeth at gastroscopy – the distance from the origin of the oesophagus is 25 cm. The three anatomic rings of the distal oesophagus are the A (muscular), B (mucosal), and C (diaphragmatic impression) rings.
[Core Radiology]
Z-lines – squamocolumnar junction
A-lines – narrowing at the origin or the vestibule of the distal oesophagus
Which structure marks the transition from squamous oesophageal to columnar gastric epithelium? [B4 Q71]
a. A-ring
b. B-ring
c. Z-line
d. oesophageal vestibule
e. gastro-oesophageal junction
The transition from squamous oesophageal epithelium to columnar gastric epithelium is
marked by the Z-line, an irregular zigzag line. It is not a reliable indicator of the gastro-
oesophageal junction, however, and may lie some distance above it if there is columnar
transformation of the distal oesophagus, as seen in Barrett’s oesophagus. The
gastrooesophageal junction may be identified by a thin, shelf-like ring known as the B-ring. It
is visible on barium swallow only when the gastro-oesophageal junction lies above the
diaphragmatic hiatus. Approximately 2–4cm above this is a thicker ring produced by active
muscle contraction known as the A-ring. The oesophageal vestibule is the saccular termination
of the lower oesophagus, which lies between the A-ring and the B-ring, and corresponds with
the lower oesophageal sphincter
A 45-year-old woman is referred by her GP for a barium swallow for investigation of dysphagia.
Gastro-oesophageal reflux into the lower third of the oesophagus is demonstrated and delicate
transverse striations in the lower oesophagus are observed as a transient phenomenon. What is
the next appropriate action appropriate for the radiologist? [B1 Q39]
A. Recommend a staging CT of chest and abdomen.
B. Recommend oesophagoscope and biopsy of the affected area.
C. Recommend to the GP that the study was unremarkable but for mild reflux.
D. Recommend referral for manometry.
E. Recommend endoscopic ultrasound.
Recommend to the GP that the study was unremarkable but for mild reflux.
The findings described are in keeping with a ‘feline’ oesophagus. This is thought to be due to spasm in the muscularis mucosa. It is associated with gastro-oesophageal reflux but is a benign entity
A neonate is diagnosed with congenital tracheoesophageal (TE) fistula. A plain film
demonstrates a gasless abdomen. Which type of TE fistula is associated with this finding? [B2
Q3]
a. Type B
b. Type C
c. Type D
d. Type E
e. None of the above
Type B
Congenital TE fistula and oesophageal atresia occur in approximately 1 in 4000 live births.
They are divided into five subtypes, A to E. Type C is the most common, comprising 75% of
all types and involves oesophageal atresia with a distal TE fistula. Type D involves
oesophageal atresia with both proximal and distal TE fistula, and type E is a TE fistula without
oesophageal atresia. Therefore, types C to E do not typically present with gasless abdomen.
Type B is oesophageal atresia with a proximal TE fistula; there is no communication between
the trachea and the distal oesophagus, and therefore a gasless abdomen is typical. Type A is
oesophageal atresia without TE fistula and therefore may also present with a gasless abdomen
but is not a listed option.
TE fistula classification [STATdx]
Type A: Oesophageal atresia with no TEF (7-9%)
Type B: Oesophageal atresia with proximal TEF (1%)
Type C: Oesophageal atresia with distal TEF (82-86%)
Type D: Oesophageal atresia with proximal & distal TEF (2%)
Type E: TEF without Oesophageal atresia (4-6%)
A 45-year-old man presents with dysphagia and undergoes a double-contrast barium swallow. This demonstrates a smooth oblique indentation on the posterior wall of the oesophagus. What is the most likely cause of these appearances? [B4 Q37]
a. enlarged left atrium
b. aberrant right subclavian artery
c. aberrant left pulmonary artery
d. right-sided aortic arch
e. coarctation of the aorta
Aberrant right subclavian artery
Several anomalies of the major vessels can cause extrinsic impressions upon the oesophagus.
The commonest aortic anomaly is a right-sided aortic arch, which produces an indentation on
the right lateral oesophageal wall in the absence of the normal left aortic arch impression. An
aberrant right subclavian artery originates from the aortic arch just distal to the left subclavian
artery, and passes upwards and to the right, behind the oesophagus, giving rise to an oblique
posterior oesophageal indentation. In aortic coarctation, the preand post-stenotic dilatations of
the aorta produce a characteristic reversed-3 impression upon the left wall of the oesophagus.
An enlarged left atrium and an aberrant left pulmonary artery both cause anterior indentations
upon the oesophagu
A 76-year-old woman presents with dysphagia and regurgitation of undigested food. She
undergoes barium swallow, which demonstrates a barium-filled pouch extending from the
posterior oesophageal wall at the level of C5–6 that is causing oesophageal compression. What
is the most likely diagnosis? [B4 Q61]
a. intramural pseudodiverticulum
b. epiphrenic diverticulum
c. lateral pharyngeal diverticulum
d. interbronchial diverticulum
e. Zenker’s diverticulum
Zenker’s diverticulum
A Zenker diverticulum is a herniation of the mucosa and submucosa through the midline of the
posterior oesophageal wall at the cleavage plane between the oblique and transverse fibres of
crico-pharyngeus (Killian’s dehiscence) at the level of C5–6. The diverticulum is narrow
necked and extends caudally, resulting in trapping of undigested food and compression of the
adjacent oesophagus. Epiphrenic diverticula are rare, usually occurring on the right lateral wall
of the distal oesophagus in association with hiatus hernia. Lateral pharyngeal diverticula are
herniations of pharyngeal mucosa through the lateral pharyngeal wall, which occur most
commonly in wind instrument players, reflecting increased intra-pharyngeal pressure. An inter-
bronchial diverticulum is a traction diverticulum that occurs in the inter-bronchial segment of
the oesophagus in response to adjacent fibrous adhesions following lymph-node infection
(usually tuberculous). Intramural pseudodiverticula represent dilated excretory ducts of
mucosal glands, which appear as multiple flask-shaped outpouchings and are commonly seen
in association with candidiasis
A 51-year-old male patient has a barium swallow for the investigation of dysphagia. This
shows a 10 cm tapered stricture in the mid oesophagus along with multiple fine linear
projections perpendicular to the lumen, each 3–4mm long, in this segment. There are
occasional tertiary contractions and mild gastro-oesophageal reflux. What is the most likely
diagnosis? [B2 Q50]
a. Chagas disease
b. Oesophageal intramural pseudodiverticulosis
c. Oesophageal varices
d. Cytomegalovirus infection
e. Oesophageal carcinoma
Oesophageal intramural pseudodiverticulosis
Oesophageal intramural diverticulosis relates to
dilated excretory ducts of the deep mucous glands
of the oesophagus. They are best demonstrated on
barium swallow and have the classical
appearance as described in the question. The
pseudo-diverticular can appear to float outside
the oesophagus when no communication with the
lumen is seen. Most patients have dysphagia at
presentation and associated conditions include
diabetes, candida infection, oesophagitis,
stricture, and alcohol abuse
Case courtesy of Dr Andres Turchetti,
Radiopaedia.org, rID: 12848
A45-year-old man undergoes barium swallow for dysphagia, which demonstrates multiple
flask-shaped outpouchings of barium arranged in longitudinal rows paralleling the long axis of
the oesophagus. Which of the following is a commonly associated condition? [B4 Q51]
a. scleroderma
b. rheumatoid arthritis
c. chronic obstructive airway disease
d. AIDS
e. diabetes
Diabetes
Oesophageal intramural pseudodiverticulosis is a condition causing dilatation of the ducts of
the submucosal glands of the oesophagus. These appear on barium meal as multiple, tiny, flask-shaped collections of barium arranged in longitudinal rows. They may appear to ‘float’ outside
the oesophagus, as the connection to the lumen may not be appreciated. Associated strictures
in the distal oesophagus are common. The condition is commonly associated with diabetes and
chronic alcoholism but may also occur with severe oesophagitis of any cause. Candida may be
cultured in around half the cases, but this may be a secondary infection due to stasis of
secretions within the glands.
A 55-year-old man presents with dysphagia. He gives no history of weight loss and
investigations reveal a normal full blood picture. He is referred for a barium swallow, which
reveals a long stricture (several centimetres) in the mid to distal oesophagus with a fine reticular
pattern adjacent to the distal aspect of the stricture and distal oesophageal widening. What is
the most likely diagnosis? [B1 Q33]
A. Reflux oesophagitis.
B. Candidiasis.
C. Barrett’s oesophagus.
D. Oesophageal adenocarcinoma.
E. Hiatus hernia.
Barrett’s oesophagus
This represents progressive columnar metaplasia of
the distal oesophagus secondary to reflux
oesophagitis. It is a premalignant condition
associated with an increased risk of adenocarcinoma,
40-fold that of the general population. Strictures are
more common in the distal, then mid oesophagus,
rather than the classically described proximal third.
The typical finding is of 1-cm- long strictures or
ulceration with associated gastro-oesophageal reflux
and hiatus hernia. These findings are non-specific
and may result from a variety of other causes such as
corrosive ingestion, nasogastric intubation, Crohn’s
disease, or neoplasm (primary or secondary).
However, the presence of a fine reticular pattern
extending distally from the stricture appears to be
specific for Barrett’s. A reticulonodular pattern has
been described in patients with a superficial
spreading adenocarcinoma, but this is rare and not
classically associated with a stricture.
Case courtesy of Dr Matt A. Morgan, Radiopaedia.org, rID: 44421
A 30-year-old man with a long history of dysphagia presents with food impaction. He has a
past medical history of allergies but nothing else of note. The food bolus passes spontaneously,
and a water-soluble followed by a barium swallow are requested prior to endoscopy, to ensure
there has been no perforation due to chicken/fishbones. The barium study reveals a moderately
long stricture in the lower oesophagus, with multiple distinct ring-like indentations. What is
the most likely diagnosis? [B1 Q56]
A. Idiopathic eosinophilic oesophagitis (IEE).
B. Crohn’s disease.
C. Oesophageal carcinoma.
D. Oesophageal perforation. E. Peptic stricture.
Idiopathic eosinophilic oesophagitis (IEE).
The cause of this condition is uncertain, but most authors believe it occurs as an inflammatory
response to ingested food allergens. A history of allergies is more closely correlated in children
with the condition than in adults. Only a minority of adults with IEE have peripheral blood
eosinophilia or eosinophilic gastroenteritis. The condition is most common in males aged 20–
40 who have a history of dysphagia and recurrent food impactions. The appearance of the
stricture, with its distinctive ring-like indentations, has been termed a ‘ringed’ oesophagus.
These indentations are characterized by multiple closely spaced concentric rings that traverse
the stricture.
A similar finding may be seen in congenital oesophageal stenosis, which typically occurs in
the same demographic group, with similar symptoms. The ‘ringed’ oesophagus is thus
relatively specific for IEE but is not a necessary finding (in the study quoted, it was only present
in 7 of the 14 patients, although these 7 all had strictures).
In peptic strictures, the fixed transverse folds are incomplete and further apart, producing a
characteristic step-ladder appearance as a result of trapping of barium between the folds.
Three different rings of distal oesophagus [Radiopaedia]
- A ring – transient, smooth muscular ring, above the vestibule (V)
- B ring (Schatzki ring if symptomatic) – thin mucosal ring at the gastro-oesophageal
junction, associated with hiatus hernia - C ring – diaphragmatic indentation, associated with hiatus hernia.
Idiopathic Eosinophilic Oesophagitis
[Radiopaedia]
- Middle-aged male
- Food/ allergen triggers eosinophilic
activation within oesophageal wall - Oesophageal strictures, webs, and spasm
- Radiology
o Ringed oesophagus
o Ring-like structures may co-exist with
long strictures – associated with oesophageal
spasm, dysmotility and foreshortening.
Two images from an esophagram in a 33-
year-old woman complaining of food sticking in her esophagus demonstrate several ring-like
strictures of the proximal esophagus (white solid arrow) as well as more distal and longer
strictures (white curved arrow). These were persistent on multiple films.
A 67-year-old man is referred for a barium swallow from the surgical outpatient department
with a history of dysphagia to solids. A mid-oesophageal stricture is demonstrated. Which one
of the following causes is unlikely to be in the differential? [B2 Q12]
a. Barrett’s oesophagus
b. Squamous cell carcinoma of the oesophagus
c. Schatzki ring
d. Caustic substance ingestion
e. Epidermolysis bullosa
Schatzki ring
All are reasonable differentials for a mid-oesophageal stricture, albeit with varying degrees of
frequency, except for a Schatzki ring which is found in the lower oesophagus. It occurs near
the squamocolumnar junction and is associated with reflux. It is non distensible and best seen
in the prone position on barium swallow examinations. Schatzki rings are often asymptomatic,
but oesophageal dilatation may be required where dysphagia is severe.
A 74-year-old female patient undergoes a barium swallow and meal as part of investigation of
anaemia, as she refuses endoscopy. She denies any weight loss, dysphagia, or odynophagia.
The swallow reveals multiple rounded plaques and nodules in the mid oesophagus. What is the
most likely diagnosis? [B1 Q35]
A. Oesophageal candidiasis.
B. Herpes oesophagitis.
C. HIV oesophagitis.
D. Glycogenic acanthosis.
E. Cytomegalovirus oesophagitis.
Glycogenic acanthosis
Glycogenic acanthosis is a common condition affecting elderly people. Cytoplasmic glycogen
accumulates in the squamous epithelial lining of the oesophagus, producing the findings
described in the question. Patients usually have no oesophageal symptoms, and the disease is
not a precursor of malignancy (although extensive glycogenic acanthosis has been shown to be
associated with Cowden’s syndrome). The major differential diagnosis is candidiasis, but the
plaques of candidiasis plaques have a more linear, rather than rounded, appearance and it usually occurs
in immunocompromised patients who complain of odynophagia. Options C, D, and E typically
cause ulceration, not plaques.
A 26-year-old man known to have AIDS presents with a 2-week history of difficult and painful
swallowing. He undergoes double-contrast barium examination of the oesophagus, which
demonstrates multiple, small, superficial, round ulcers in the mid-oesophagus. The intervening
mucosa is normal, and no plaques are seen. What is the most likely diagnosis? [B4 Q34]
a. HIV oesophagitis
b. cytomegalovirus oesophagitis
c. reflux oesophagitis
d. candida oesophagitis
e. herpes simplex oesophagitis
Herpes simplex oesophagitis
Candida oesophagitis is the commonest cause of infectious oesophagitis and is particularly
seen in immunosuppressed individuals. It is frequently associated with oral thrush. It tends to
affect the upper half of the oesophagus, and typical appearances are of linear, longitudinally
oriented filling defects representing heaped-up areas of mucosal plaques consisting of necrotic
debris and fungal colonies. In contrast, a normal intervening mucosa in oesophagitis is
suggestive of a viral aetiology.
Cytomegalovirus and HIV oesophagitis: One large flat seen in the distal oesophagus. Distinction between the two is made by brushings or biopsy at endoscopy.
Herpes simplex infection, the typical features of multiple, small, round superficial ulcers with surrounding radiolucent halo are similar at all sites of potential involvement, including the oesophagus, oral cavity, rectum, and anus.
A 32-year-old male is referred for a barium swallow by his GP due to dysphagia resistant to
medical treatment. A smooth, lobulated, eccentric mass is seen in the middle third of the
oesophagus containing foci of calcification. The diagnosis is most likely to be which one of
the following? [B2 Q10]
a. Leiomyoma
b. Squamous cell carcinoma
c. Adenocarcinoma
d. Oesophageal web
e. Intramural pseudodiverticulosis
Leiomyoma
Oesophageal leiomyoma is the most common benign submucosal
tumour of the oesophagus, typically occurring in young men. The
classical features of oesophageal leiomyoma include a smooth
intramural mass in the lower or middle third of the oesophagus with
intact overlying mucosa. It is the only tumour of the oesophagus that
calcifies, although calcification is rare.
Case courtesy of Dr Mohammadtaghi Niknejad, Radiopaedia.org, rID:
62519
A 23-year-old man with dysphagia undergoes a double-contrast
barium swallow, which demonstrates a smooth, well-defined, 12 cm submucosal lesion in the
distal oesophagus causing deformity of the lumen. CT demonstrates coarse calcification within
the mass. What is the most likely diagnosis? [B4 Q22]
a. oesophageal lipoma
b. oesophageal duplication cyst
c. oesophageal carcinoma
d. oesophageal varices
e. oesophageal leiomyoma
Oesophageal leiomyoma
Leiomyomas are benign tumours of smooth muscle and represent the most common benign
neoplasm of the oesophagus. They are often asymptomatic but may present with dysphagia and
rarely haematemesis. They appear on barium swallow as large, well-defined, intramural masses
causing luminal deformity. A characteristic finding is of coarse calcifications – leiomyoma is
the only calcifying oesophageal tumour. Oesophageal lipomas and duplication cysts also
appear as well-defined submucosal lesions (of fat and of water density respectively on CT), but
are less common, and internal calcification is not a feature. Oesophageal carcinoma usually
appears as an irregular ulcerated stricture. Oesophageal varices are seen as serpiginous filling
defects.
What is the most common cause of varices affecting the upper third of the oesophagus? [B4
Q52]
a. portal hypertension due to cirrhosis
b. splenic vein thrombosis
c. inferior vena caval obstruction
d. superior vena caval obstruction
e. hepatic vein obstruction
Superior vena caval obstruction
Oesophageal varices are dilated submucosal veins, which may be classified by their direction
of flow as uphill or downhill varices. Uphill varices occur in the lower oesophagus and
represent collateral blood flow conveying portal venous blood to the azygos vein. They usually
result from portal hypertension due to liver cirrhosis, but may also occur with splenic vein
thrombosis, and obstruction of the hepatic veins or IVC. Downhill varices result from
obstruction of the SVC. If it is obstructed superior to the entry of the azygos vein, varices will
be confined to the upper third of the oesophagus. If the SVC is obstructed below the entry of
the azygos vein, the varices convey all the systemic venous blood from the upper half of the
body into the portal vein and IVC, and they will run the entire length of the oesophagus. SVC
obstruction is most commonly due to lung cancer or lymphoma.
Which of the following are CT criteria for T3 rather than T2 oesophageal cancers? [B3 Q30]
A. Depth of 11mm
B. Transmural enhancement
C. Focal wall thickening measuring 14mm
D. A few < 1⁄3 small linear strands of soft tissue extending into fat planes
E. Slight stenosis
Depth of 11mm
T3 disease usually involves a large tumour more than 10 mm in depth where > 1 ⁄ 3 tumour
extension or a blurred fat plane around the lesion is associated with moderate/severe stenosis.
Focal wall thickening of 5-15mm is still T2 disease.
Oesophageal Tumour Staging [Radiopaedia]
- T1 – Before muscularis propria
o T1a – Lamina propria and muscularis mucosae
o T1b – submucosa - T2 – Muscularis propria
- T3 – Adventitia
- T4 – Adjacent Structures
o T4a – pleura, pericardium, azygous vein, diaphragm, peritoneum
o T4b – others s/s aorta, vertebra1
At endoscopic ultrasound scan for staging of an oesophageal carcinoma, the tumour is seen
extending into the hypoechoic fourth layer of the oesophagus but not beyond this. What is the
T staging of the tumour? [B4 Q94]
a. Tis
b. T1
c. T2
d. T3
e. T4
T2
Endoscopic ultrasound is the most accurate method for local staging of oesophageal cancer. At
endoscopic ultrasound, the oesophageal wall appears as five distinct alternating hyperechoic
and hypoechoic bands that correspond to the histological layers of the oesophagus.
The innermost hyperechoic layer represents the interface between the lumen and the mucosa.
The second layer is a hypoechoic band that represents the muscularis Mucosa.
The third layer is a hyperechoic band that represents the submucosa.
The fourth layer is a hypoechoic band that represents the muscularis Propria.
The fifth outermost layer is a hyperechoic band that represents the oesophageal adventitia.
The fifth layer in the stomach, duodenum and rectum represents the serosa. For oesophageal cancer, T1 tumours invade the lamina propria or submucosa. T2 tumours invade the muscularis propria, T3 tumours invade the adventitia and T4 tumours invade adjacent tissue. Tis represents carcinoma in situ*
A 65-year-old man undergoes endoscopy for dysphagia, during which an ulcerated mass is
seen in the distal oesophagus. Biopsy confirms oesophageal adenocarcinoma. What is the most
accurate imaging modality for local staging of oesophageal cancer? [B4 Q3]
a. endoscopic ultrasound
b. CT
c. 18FDG PET/CT
d. MRI
e. barium swallow
Endoscopic ultrasound
CT is the most used imaging investigation for staging of oesophageal cancer. However, the
overall accuracy of T staging is poor, particularly with T1 and T2 tumours, and CT also tends
to overestimate tumour length. Endoscopic ultrasound scan is the most accurate imaging
method for local staging but is limited in its assessment of nodal and metastatic disease. 18
FDG PET/CT is useful in evaluation of nodal and metastatic disease, particularly in patients
being considered for surgical resection, but has limited resolution for T staging and often fails
to demonstrate T1 lesions. MR is useful in characterization of indeterminate liver lesions seen
on CT. Barium swallow is not used in the staging of oesophageal cancer.
A 50-year-old woman presents with dysphagia. At barium swallow, contrast passes sluggishly
into the oropharynx. No peristaltic waves are seen in the upper oesophagus. After swallowing,
the lumen of the hypopharynx and upper oesophagus remain patent and distended. The lower
oesophagus outlines normally. What is the most likely diagnosis?
A. Achalasia.
B. Scleroderma.
C. Polymyositis.
D. Chagas disease.
E. SLE.
Polymyositis
This condition and dermatomyositis affect skeletal muscle, which is found at the upper third
of the oesophagus. These conditions begin in the upper oesophagus and extend caudally. Other
findings at fluoroscopy include retention of barium in the valleculae and wide atonic pyriform
fossae, regurgitation and nasal reflux, aspiration, and failure of contrast to progress in the upper
oesophagus without the aid of gravity. Polymyositis and dermatomyositis are associated with
underlying malignancy. The latter also involves a heliotrope rash and Gottron’s papules on
flexor surfaces.
The lower oesophagus is composed of smooth muscle and is affected by conditions such as
scleroderma and SLE, which result in atony and lack of peristalsis in the lower two-thirds,
beginning caudally and moving cranially. Achalasia and Chagas disease result in dilatation of
the whole oesophagus, with a ‘rat-tail’ deformity at the lower end.
A 71-year-old female with scleroderma undergoes a barium swallow examination. Which one
of the following findings concerning the oesophagus would not be consistent with this
diagnosis? [B2 Q1]
a. Oesophageal dilatation
b. Superficial ulcers
c. Hypoperistalsis in the upper third of the oesophagus
d. Stricture 5cm above the gastro-oesophageal junction
e. Oesophageal shortening
Hypoperistalsis in the upper third of the oesophagus
The oesophagus is the most involved location of the gastro-intestinal tract in patients with
scleroderma. Smooth muscle atrophy causes hypoperistalsis and eventually aperistalsis in the
lower two-thirds of the oesophagus. The upper third of the oesophageal wall contains skeletal
muscle and is therefore unaffected by the disease process.
A 46-year-old woman with a multisystem disorder presents with dysphagia and heartburn.
Barium swallow reveals a dilated oesophagus with aperistalsis of the lower two-thirds of the
oesophagus, a patulous lower oesophageal sphincter and gastro-oesophageal reflux. Which
other organ system is most likely to be affected? [B4 Q1]
a. respiratory
b. cardiovascular
c. skin
d. central nervous
e. renal
Skin
The patient is suffering from systemic sclerosis, a multisystem connective tissue disorder of
unknown aetiology, classified by extent of skin involvement and overlap with other
autoimmune disorders. The skin is the most involved organ, demonstrating thickening, atrophic
changes and fibrosis. The gastrointestinal system is the next most affected, with around 50%
of patients having symptomatic disease. The oesophagus is most frequently involved, with
fibrosis of the circular layer of smooth muscle resulting in a dilated oesophagus with absent or
reduced peristalsis in the lower two-thirds. The lower oesophageal sphincter is wide, in contrast
to the tapered narrowing seen in achalasia. Patients suffer from reflux that predisposes to
Barrett’s oesophagus and distal strictures. The cardiovascular, respiratory, central nervous and
renal systems may all be affected in systemic sclerosis, though less commonly than the skin
and gastrointestinal systems.
A 47-year-old woman with dysphagia undergoes barium swallow, which demonstrates a
persistent smooth posterior bulge at the pharyngo-oesophageal junction at the level of C5–6
with mild proximal pharyngeal dilatation. What is the most likely diagnosis? [B4 Q97]
a. normal findings
b. impaired crico-pharyngeus relaxation
c. pharyngeal web
d. anterior cervical osteophytes
e. thyroid enlargement
Imparid crico-phargyneus relaxation
Impaired cricopharyngeus relaxation (or cricopharyngeal achalasia) is hypertrophy of the
cricopharyngeus muscle with failure of relaxation. It is seen in up to 10% of asymptomatic
adults as a normal variant, as a compensatory mechanism in gastro-oesophageal reflux, and in
association with a range of neuromuscular disorders. It appears on barium swallow as a smooth,
shelf-like posterior projection at the level of C5–6 that persists during a swallow. In severe
cases, it may result in functional obstruction or overflow aspiration. Symptomatic patients may
be treated by cricopharyngeal myotomy.
Pharyngeal webs are thin, anterior, shelf-like protrusions into the cervical oesophagus. They
are frequent incidental findings but occasionally cause dysphagia. There is an association with
Plummer–Vinson syndrome. Anterior osteophytes may cause an indentation of the posterior
oesophagus, but these are usually asymptomatic. Thyroid enlargement may cause a smooth
impression on the lateral wall of the oesophagus.
A 70-year-old man with a history of several months of dysphagia undergoes double-contrast
barium swallow. This demonstrates a moderately dilated oesophagus with reduced peristalsis
and smooth tapering of the distal oesophagus. What is the most likely diagnosis? [B4 Q19]
a. primary achalasia
b. gastric carcinoma
c. scleroderma
d. oesophageal carcinoma
e. presbyoesophagus
Gastric carcinoma
Primary achalasia is an abnormality of the myenteric plexus resulting in reduced or absent
peristalsis and failure of relaxation of the lower oesophageal sphincter. The oesophagus is
typically markedly dilated with absent primary peristalsis and a smooth tapered narrowing at
the contracted lower oesophageal sphincter. It usually presents in young adults with long-
standing dysphagia. In contrast, secondary achalasia due to malignancy usually presents in
older patients with a short duration of dysphagia. Decreased peristalsis and distal oesophageal
tapering in these patients result from tumour infiltration of the myenteric plexus of the distal
oesophagus by gastric carcinoma, lymphoma, or metastatic disease. Distal oesophageal
carcinoma tends to give rise to irregular, asymmetrical narrowing. Scleroderma typically
appears as a dilated oesophagus with a patulous lower oesophageal sphincter.
Presbyoesophagus is a disorder of oesophageal motility, characterized by oesophageal
dilatation and repetitive, non-peristaltic, tertiary contractions in the distal oesophagus.
A 75-year-old woman presents with severe chest pain radiating to her back and some
haematemesis. The surgical team have considered a differential diagnosis of aortic dissection
or aorto-enteric fistula and requested a CT scan to assess the aorta. No aortic dissection is seen,
but there is a long eccentric filling defect identified within the oesophageal wall, extending
from the level of the carina to the gastro-oesophageal junction. This area did not enhance after
contrast administration but did measure 75 HU on a pre-contrast scan. Barium swallow
revealed a longitudinal impression on the oesophagus, which had resolved on a repeat swallow
6 weeks later. What is the most likely diagnosis? [B1 Q36]
A. Aorto-oesophageal fistula.
B. Mallory–Weiss tear.
C. Boerhaave syndrome.
D. Oesophageal varices.
E. Intramural haematoma of the oesophagus.
Intramural haematoma of the oesophagus
Submucosal dissection of the oesophagus may be spontaneous or secondary to direct trauma
or coagulopathy. Patients may present with chest pain, dysphagia, and nausea, often followed
by haematemesis. The high attenuation in the wall of the oesophagus is the clue to the diagnosis.
This feature and lack of enhancement are inconsistent with any alternative diagnosis. Follow-
up with endoscopy is usually performed to exclude a predisposing pathological condition. The
natural history is complete resolution without surgical intervention.
Which of the following represent a case of oesophageal pouch in the lower 1⁄3 of the
oesophagus? [B3 Q37]
A. Mucosal tear/Mallory-Weiss
B. Intraluminal diverticulosis
C. Defect through Killian-Jamieson space
D. Zenker’s diverticulosis
E. Traction following treatment for TB
Mucosal tears from Mallory-Weiss syndrome or post endoscopy are typically in the lower 1/3
of the oesophagus.
A 56-year-old man presents acutely with chest pain after a night out. On examination he is
febrile, tachycardic and hypotensive. Chest radiograph demonstrates extensive
pneumomediastinum, with left pleural effusion and left lower lobe atelectasis. What is the most
likely diagnosis? [B4 Q24]
a. acute pulmonary embolism
b. spontaneous oesophageal rupture
c. aortic dissection
d. lobar pneumonia
e. acute pancreatitis
Spontanous oesophageal rupture
Iatrogenic injury is the most common cause of oesophageal rupture, but, in 15% of cases,
rupture is spontaneous and occurs during vomiting (Boerhaave’s syndrome). Patients present
with pain and dysphagia, and rapidly develop sepsis. Characteristic chest radiograph findings
are of extensive pneumomediastinum and subcutaneous emphysema, pleural effusion or
hydropneumothorax, and left lower lobe atelectasis. Widening of the mediastinum may
accompany the development of mediastinitis. Pleural effusion and atelectasis may be seen in
acute pulmonary embolism, acute pancreatitis, and aortic dissection, but pneumomediastinum
is not a recognized feature of these conditions. Other common causes of pneumomediastinum
include asthma, chest trauma and perforation of a hollow viscus with extension of gas via the
retroperitoneum.
A patient is undergoing a Ba meal. What is the best position to place the patient in to see an
enface view of lesser curvature? [B1 Q13]
A. Left lateral
B. Left anterior oblique (LAO)
C. Supine
D. Right anterior oblique (RAO)
E. Right lateral
LAO: Lesser curvature.
RAO: Body and Antrum.
Supine: Greater curve and Antrum.
Left lateral: Fundus.
You are left in charge of a barium meal list. Due to an acute staff shortage, there is only a
student radiographer with you, who wants to know about which barium to use and why. Which
one of the following statements regarding barium contrast media is correct? [B1 Q19]
A. Simethicone is added to reduce flocculation.
B. The weight/volume ratio of barium for barium meals is 150%.
C. The same weight/volume ratio is used for barium meals and follow-through examinations.
D. Uniform particle size improves mucosal coating.
E. Gastrografin can be added to improve transit time.
Gastrografin can be added to improve transit time
Simethicone is an antifoaming agent. While a uniform particle size helps reduce flocculation,
a heterogeneous particle size improves mucosal coating. The barium densities used for
different examinations are barium swallow 150%, barium meal 250%, barium follow-through
50%, barium small bowel enteroclysis 18%, double-contrast barium enema 125%, and single-
contrast barium enema 70%.
Which is the most appropriate contrast medium for a barium follow-through examination of
the small bowel? [B4 Q65]
a. 100 ml of 250% w/v barium sulphate
b. 135 ml of 250% w/v barium sulphate
c. 300 ml of 100% w/v barium sulphate
d. 1500 ml of 20% w/v barium sulphate
e. 500 ml of 115% w/v barium sulphate
300 ml of 100% w/v barium sulphate
The recommended contrast is 300ml of 100% w/v barium sulphate. Transit time may be
reduced by the addition of 10ml of Gastrografin to the barium. Non-ionic, water-soluble
contrast media may be used as an alternative where barium is contraindicated. The
recommended concentrations and volumes for the other gastrointestinal contrast examinations
are barium swallow (a), barium meal (b), small bowel enema (d) and double-contrast barium
enema (e).
A six-week-old child has an ultrasound scan of the abdomen performed for non-bilious
projectile vomiting. Which one of the following features would support a diagnosis of infantile
pylorospasm over a diagnosis of hypertrophic pyloric stenosis? [B1 Q36]
a. Pyloric muscle wall thickness of 2 mm
b. Pyloric canal length of 19 mm
c. Target sign
d. Antral nipple sign
e. Transverse pyloric diameter of 14 mm
Pyloric muscle wall thickness of 2 mm
Hypertrophic pyloric stenosis presents between four and six weeks of life with non-bilious
vomiting, typically in first-born males. A palpable olive-shaped mass is a sign with reported
sensitivity of up to 80%, but ultrasound is the most frequently used imaging modality. Typical
ultrasound features include the target sign (central hyperechoic mucosa with surrounding
hypoechoic pyloric muscle), the nipple sign (pyloric mucosa indenting the gastric antrum),
pyloric canal length > 16 mm, transverse pyloric diameter >13 mm and pyloric muscle wall
thickness > 3 mm. Pyloric stenosis can be difficult to differentiate radiologically from infantile
pyloro-spasm. Typically, with pyloro-spasm the appearances change with time, and so if the
pyloric muscle thickness is measured at less than 3 mm this makes infantile pyloro-spasm the
more likely diagnosis.
Hypertrophic Pyloric Stenosis [STATdx]
- Idiopathic pyloric muscle thickening in young infants causing progressive gastric outlet
obstruction - Near-complete gastric outlet obstruction due to abnormally elongated & thickened pyloric
muscle - Pathology
- Idiopathic, prostaglandin or erythromycin induced, neural mediated, familial
- Tends to run in families.
- Clinical Features
- 2-12 weeks old, progressive, nonbilious projectile vomiting
- Feedings previously tolerated
- Male preponderance, common
- Gastroesophageal reflux > > HPS
- Ultrasound * Hypertrophied circumferential hypoechoic muscle & elongated pyloric canal filled with
echogenic mucosa - Pyloric channel length > 15-16 mm
- Single wall thickness of pyloric muscle > 3 mm
- Failure of thickened pylorus to change during exam in contrast to pylorospasm.
- Fluoroscopy
- Minimal barium traversing narrowed & elongated pyloric channel
- Mass effect on gastric antrum & duodenum by hypertrophied muscle, hyperperistaltic
gastric contractions, and - Gastroesophageal reflux/emesis
- HPS in 1/5 of infants imaged for vomiting
On a barium meal examination, the incisura angularis marks the border between which
structures? [B4 Q76]
a. lesser and greater curvatures of the stomach
b. antrum and pylorus of the stomach
c. fundus and body of the stomach
d. body and antrum of the stomach
e. oesophagus and the stomach
Body and antrum of the stomach
The stomach is divided into the fundus, body, antrum, and pylorus. The fundus is that part of
the stomach extending superiorly and to the left of the cardiac orifice. The body extends from
the cardiac orifice to the incisura angularis, which is a constant notch at the lower end of the
lesser curvature marking the border between the body and the antrum of the stomach. The
antrum extends from the incisura angularis to the proximal pylorus.
At abdominal ultrasound scan, when scanning the abdomen in a transverse plane at the level
of the pancreas, which of the following structures may normally be seen lying between the
superior mesenteric artery and the aorta? [B4 Q77]
a. splenic vein
b. left renal vein
c. neck of the pancreas
d. inferior vena cava
e. common bile duct
Left renal vein
Ultrasound scan of the pancreas may be difficult and vascular landmarks are important in its
identification. In the transverse plane, the splenic vein can be seen coursing from the splenic
hilum towards the liver, and the body and tail of the pancreas lie immediately anterior to this.
The neck of the pancreas lies immediately anterior to the confluence of the splenic and superior
mesenteric veins, and the head and uncinate process of the pancreas lie around this confluence,
anterior to the inferior vena cava. At this level, the left renal vein can be seen entering the
inferior vena cava, passing between the superior mesenteric artery and aorta
A 50-year-old male is admitted with epigastric pain, diarrhoea, and vomiting. Ascites is present
clinically. Serum albumin is low and the patient is anaemic. Colonoscopy is normal but the
patient is intolerant of upper gastro-intestinal endoscopy. Barium meal reveals a normal antrum
but elsewhere there are diffusely thickened and enlarged gastric folds despite good gastric
distension. Which one of the following is the most likely diagnosis? [B2 Q26]
a. Gastric lymphoma
b. Menetrier’s disease
c. Gastric adenocarcinoma
d. Acute gastritis
e. Linitis plastica
Menetrier’s disease
Menetrier’s disease is a condition characterised by gastric mucosal hypertrophy and protein-
losing enteropathy. It is often associated with anaemia. The changes are most marked along the
greater curve and the antrum is spared in approximately 50% of cases.
Gastric lymphoma typically involves the antrum.
Gastric adenocarcinoma and linitis plastica, stomach distension is not typically preserved.
A 48-year-old man presents with epigastric pain, weight loss and peripheral oedema. Blood
tests demonstrate hypo-albuminaemia. At barium meal the stomach is well distended, but there
is poor mucosal coating. Markedly enlarged and tortuous gastric rugae are seen in the fundus
and body of the stomach, with sparing of the antrum. What is the most likely diagnosis? [B4
Q75]
a. lymphoma
b. Menetrier’s disease
c. gastric carcinoma
d. Zollinger–Ellison syndrome
e. eosinophilic gastroenteritis
Menetrier’s disease
Meneterier’s disease is characterized by mucosal hypertrophy of the fundus and body of the
stomach, with excessive mucus secretion and a protein-losing enteropathy. There may be
associated gastric ulceration. Barium meal shows impaired mucosal coating due to
hypersecretion and marked gastric fold thickening, though the stomach distends normally. The
stomach is the commonest site for gastrointestinal lymphoma, which may be polypoid,
ulcerating or infiltrative. The infiltrative form may cause pronounced thickening of gastric folds,
with preserved stomach distensibility, but hypersecretion is not a feature. Infiltrating gastric
carcinoma may also cause thickened gastric folds, but associated desmoplastic reaction results in a rigid, poorly distensible stomach. Zollinger–Ellison syndrome results in hypersecretion of
gastric acid which impairs mucosal coating of barium and is associated with ulceration and
enlargement of rugal folds, but hypoproteinaemia is not a feature. Eosinophilic gastroenteritis
may cause enlarged gastric folds and be associated with protein-losing enteropathy if the small
bowel is involved. However, the antrum is most involved.
A 65-year-old woman, with a history of previous partial gastrectomy 10 years earlier, presents
with upper abdominal pain and early satiety. She undergoes a double-contrast barium meal,
which demonstrates a 4 cm intraluminal, mottled filling defect in the gastric remnant with no
fixed attachment to the gastric wall. What is the most likely diagnosis? [B4 Q21]
a. suture granuloma
b. trichobezoar
c. phytobezoar
d. gastric carcinoma
e. villous adenoma
Phytobezoar
Bezoars are masses of accumulated ingested material forming in the stomach or intestines.
Phytobezoars are the commonest type, composed of poorly digested fibre and vegetable matter.
They are seen particularly in patients with previous gastric surgery, probably due to diminished
gastric emptying. Patients may be asymptomatic or present with early satiety or symptoms of
gastritis, as phytobezoars are irritant. Occasionally, they may obstruct the stomach with a ball–
valve mechanism. They are seen as relatively mobile filling defects, the interstices of which
are filled with barium. Trichobezoars are composed of hair, and are usually larger, and found
in younger patients, particularly those with a psychiatric history. Gastric carcinoma, villous
adenoma and suture granuloma are all causes of gastric filling defects but have a constant
relationship to the gastric wall
A 65-year-old man presents with early satiety and bloating and undergoes barium meal. This
demonstrates a smoothly marginated, 15 cm mass in the body of the stomach, making an obtuse
angle with the gastric wall. CT demonstrates peripheral enhancement of the mass with central
areas of low attenuation and extra-gastric extension into the lesser sac. There is no associated
lymphadenopathy. What is the most likely diagnosis? [B4 Q99]
a. gastrointestinal stromal tumour
b. gastric carcinoma
c. gastric lymphoma
d. adenomatous polyp
e. gastric carcinoid
GIST
Gastrointestinal stromal tumours are the commonest mesenchymal tumours of the
gastrointestinal tract. They are characterized by expression of KIT, a tyrosine kinase growth
factor receptor, which distinguishes them from leiomyomas and leiomyosarcomas. They occur
most commonly in the stomach and have the classic appearance of a submucosal mass on
barium meal, forming an obtuse angle with the gastric wall in profile. Focal areas of ulceration
are seen in 60%. On CT, the tumours measure up to 30 cm and are often predominantly extra-gastric. Typical features are of peripheral enhancement, with central low attenuation representing necrosis, haemorrhage, and cyst formation. NO Lymphadenopathy.
Gastric carcinoma and lymphoma rarely demonstrate exophytic growth and commonly have associated lymphadenopathy. Adenomatous polyps are mucosal lesions. Gastric carcinoid is usually seen in the antrum and characteristically shows associated ulceration.
Which of the following favours gastric lymphoma rather than other gastric malignancies? [B3
Q45]
A. Preservation of the fat plane around the stomach
B. Luminal narrowing
C. Involvement of the proximal half of the stomach
D. Heterogenous gastric wall thickening
E. A single site of disease within the stomach
Preservation of the fat plane around the stomach, diffuse and homogenous wall thickening,
multifocal disease within the stomach, nodal disease either side of the mesenteric vessels, nodal
disease extending below the level of the renal veins and a propensity for the distal half of the
stomach are all features of gastric lymphoma.
Which of the following is the most correct statement with regards to Gastrointestinal Stromal
tumour (GIST) of the stomach? [B3 Q46]
A. Most patients present below the age of 50
B. GISTS of the stomach tend to have a more aggressive histology when compared with GISTS
from other sites
C. Mural calcification is a common feature
D. Larger lesions tend to be more homogenous in enhancement
E. Central fluid attenuation/necrosis is common
Central fluid attenuation/ necrosis is common
Stomach GISTs tend to present in patients over 50. CT usually shows a well-defined
heterogeneously enhancing, round, exophytic mass, commonly with central necrosis. Mural
calcification is recognised but not common. Stomach GISTs tend to be less aggressive
histologically than GISTs at other sites.
A 54-year-old man with known metastatic malignant melanoma presents with epigastric pain
and haematemesis. What is the most likely finding in the stomach on double-contrast barium
meal? [B4 Q23]
a. multiple submucosal nodules with central ulceration
b. solitary ulcerated mass in the gastric antrum
c. linitis plastica
d. solitary, well-defined, pedunculated filling defect
e. thickened tortuous gastric folds
Multiple submucosal nodules with central ulceration
GI tract metastases are seen in 4–8% of patients with malignant melanoma. The small intestine
is most affected, followed by the colon and stomach. Typical features are of multiple
submucosal nodules, with a target appearance due to central ulceration. This appearance is
particularly seen with malignant melanoma metastases but may also be seen with gastric
metastases from breast, lung, and renal cell carcinoma. Other common appearances of gastric metastases include linitis plastica in 20%, most typically from breast cancer, and a solitary mass in 50%.
A 69-year-old man undergoes staging of gastric carcinoma diagnosed at upper gastrointestinal
endoscopy. CT of the abdomen demonstrates focal gastric wall thickening with extension into
the perigastric fat, but no invasion of adjacent structures. Five local lymph nodes measuring
10–12 mm in short axis diameter is identified. There is no distant metastatic disease. What is
the TNM staging of the tumour? [B4 Q89]
a. T2 N0 M0
b. T2 N1 M0
c. T2 N2 M0
d. T3 N1 M0
e. T3 N2 M0
T3 N2 M0
T3 tumours penetrate the subserosa but do not invade adjacent structures. On CT, this may be
appreciated as blurring of the tumour margin or wide reticular strands radiating from the
tumour edge. Nodal staging depends on the number of regional nodes visible, with nodes larger
than 8mm being regarded as pathological. The presence of 1–6 regional nodes results in a stage
of N1, with 7–15 nodes and >15 nodes representing nodal stages of N2 and N3 respectively.
Non-regional nodes such as para-aortic and retro-pancreatic nodes are considered M1 disease
A 66-year-old woman with a known large para-oesophageal hiatus hernia presents with sudden
onset of severe epigastric pain and vigorous retching without production of vomitus. Passage
of a nasogastric tube is unsuccessful. Plain abdominal radiograph demonstrates a markedly
distended stomach in the left upper quadrant extending into the chest. What is the most likely
diagnosis? [B4 Q53]
a. pyloric stenosis
b. ‘cup-and-spill’ stomach
c. acute gastric volvulus
d. acute gastric dilatation
e. paraduodenal hernia
Acute gastric volvulus
Acute gastric volvulus is abnormal rotation of one part of the stomach around another part,
which may be classified as organoaxial, mesenteroaxial or combination type, depending on the
axis of rotation. Predisposing factors include ligamentous laxity, hiatus hernia and
diaphragmatic eventration. The classic presentation is with the Borchardt triad of sudden severe
epigastric pain, intractable retching with no vomitus produced, and inability to pass a
nasogastric tube into the stomach. Other plain film findings include unexpected location of the
gastric bubble and air–fluid levels in the mediastinum or upper abdomen, but definitive
diagnosis is by barium meal. The condition is a surgical emergency, as it may result in gastric
ischaemia or perforation. Acute gastric dilatation and pyloric stenosis may result in gastric
distension on plain film but would not present with intractable retching or difficulty with
nasogastric tube passage. A ‘cup-and-spill’ stomach is an anatomical variant on barium meal,
which may simulate an organoaxial volvulus. A para-duodenal hernia usually presents acutely
as small bowel obstruction.
Gastric Volvulus [Radiopaedia]
Case courtesy of Dr Maxime St-Amant, Radiopaedia.org, rID: 19
A patient presents to an outpatient barium meal list with a history of epigastric discomfort and
weight loss of 8 kg over 6 months. The barium meal reveals an ulcer on the greater curve of
the stomach, near the pylorus. This ulcer has a surrounding mound. It is demonstrated to project
slightly beyond the lumen of the stomach. There is a thin line noted which crosses the base of
the ulcer and a degree of retraction of the greater curve around the ulcer. What type of ulcer is
this likely to be? [B1 Q6]
A. Benign due to the line noted crossing the base of the ulcer.
B. Benign due to the ulcer projecting beyond the lumen of the stomach.
C. Benign due to the surrounding mound.
D. Malignant due to the finding of scar retraction of the greater curve.
E. Malignant due to being found on the greater curve.
Benign due to the line noted crossing the base of the ulcer - Hampton’s line
This line—Hampton’s line—represents undermining of the mucosa by the more vulnerable
submucosa. It is not commonly seen but is taken to be virtually diagnostic of a benign ulcer
when present. Projection beyond the lumen and a symmetrical mound are features of a benign
ulcer along with smooth radiating mucosal folds. Scar retraction can be seen with benign ulcers.
Both benign and malignant ulcers are more commonly seen on the lesser curve.
Benign Vs Malignant Ulcers [Core Radiology]
Benign gastric ulcer
o Radiating gastric folds are smooth and symmetric
o Ulcer extends beyond the normal contour of the gastric lumen.
o The Hampton line represents nonulcerated acid-resistant mucosa surrounding the ulcer
crater.
o Most benign ulcers occur along the lesser curvature of the stomach, although benign ulcers
associated with aspirin ingestion can occur in the greater curvature and antrum, which are
dependent locations.
Malignant
o Asymmetric ulcer crater, with surrounding nodular tissue
o Abrupt transition between normal gastric wall and surrounding tissue.
o Ulcer crater does not project beyond the expected location of gastric wall.
o The Carman meniscus sign is considered pathognomonic for tumour. It describes the
splaying open of a large, flat malignant ulcer when compression is applied.
A64-year-old man undergoes a barium meal examination for upper abdominal pain. A 10 mm
ulcer is demonstrated at the gastric antrum. Which radiological feature would favour a
diagnosis of malignant rather than benign gastric ulcer? [B4 Q4]
a. round ulcer shape
b. ulcer crater confined within the gastric contour
c. gastric folds identified up to the edge of the ulcer crater
d. associated duodenal ulcer disease
e. uniform mucosal collar around a centrally located ulcer
Ulcer crater confined within gastric contour
Many distinguishing features of gastric ulceration have been proposed to classify gastric ulcers
as benign or malignant, but there is significant overlap between the two categories. One reliable
sign of a benign ulcer is the projection of the ulcer outside the gastric contour in profile, due to
excavation into the mucosal wall. In contrast, a malignant ulcer occurring within a tumour mass
does not usually extend beyond the confines of the gastric wall. Other features indicative of
benignity include a round, centrally located ulcer with a uniform collar of oedematous mucosa,
gastric folds extending to the edge of the ulcer crater and associated duodenal ulcer disease.
A 63-year-old man is day 7 post-operative following a Billroth II partial gastrectomy for a
gastric carcinoma. The initial post-operative phase was uncomplicated, but the patient has
begun complaining of increasing abdominal pain. Inflammatory markers have increased with
white cell count (WCC), rising from 12 to 42, and CRP increased from 8 to 56. A CT scan
carried out with oral and intravenous contrast demonstrates no evidence of contrast leakage
into the peritoneum. A skiff of free air is noted in the abdomen. A fluid collection is noted in
the right subhepatic space, which extends toward the peripancreatic area. What is the most
likely diagnosis? [B1 Q1]
A. Leakage from the gastroduodenal anastomosis site.
B. Leakage from the duodenal stump.
C. Post-operative pancreatitis.
D. Tumour recurrence.
E. Pseudocyst formation following post-operative pancreatitis.
Leakage from the duodenal stump.
There is no gastro-duodenal anastomosis in a Billroth II procedure. The amylase is not
sufficiently elevated for pancreatitis in most cases and there is no described abnormality in the
pancreas. It is too early for pseudocyst formation and tumour recurrence.
Different Surgeries for Peptic Ulcer Disease
- Billroth I partial gastrectomy – Seldom performed
* Distal gastrectomy
* Gastro-duodenal anastomosis (proximal stomach to D1) - Billroth II/ Polya partial gastrectomy
* Distal gastrectomy
* Closure of duodenal stump
* Gastro-jejunal anastomosis (retro or antecolic, iso or anti-peristaltic direction) - Subtotal gastrectomy with Roux Loop reconstruction
* Distal gastrectomy
* Closure of duodenal stump
* Continuity is restored with a Roux loop reconstruction - Total gastrectomy (in Ca stomach)
* Total gastrectomy with lymphadenectomy
* Duodenal stump is stapled or over sewn
* A Roux loop is anastomosed to distal oesophagus (end to side) with blind limb
* Jejuno-jujunal anastomosis 50-60 cm distal to oesophago-jejunal anastomosis - Truncal vagotomy
- Wedge Resection
- Gastro-enterostomy (to overcome the GOO)
Side to side gastro-jejunal anastomosis
No Roux loop → increased risk of bile gastritis and associated stomach cancer.
Post-operative Complications
- Anastomotic leak
- Most serious, highest after total gastrectomy
- Extra-luminal contrast leak, duodenal stump is not usually evaluated in Ba study
- Disproportionate pneumoperitoneum
- Collections in unusual place (left subphrenic space) as anatomy is disrupted
- Duodenal stump leak
- Right subhepatic space fluid collection, can be precipitated by afferent loop obstruction
- Disruption of duodenal stump metal staple line
- Afferent Loop Obstruction/Syndrome
- Associated with Billroth II gastrectomy and gastrojejunostomy
- Adhesion or twisting of the bowel proximal to gastroenterostomy site
- Acute obstruction can lead to duodenal stump blow out
- Risks – long afferent limb (>30-40 cm), antecolic gastrojejunostomy.
- Ba study is of limited value, as doesn’t normally fill the afferent limb
- CT → fluid filled dilated afferent limb with associated biliary dilatation
- Anastomotic stricture
- Parastomal jejunal ulcer
- Common after Billroth II as gastric acid goes into jejunum.
- Very rarely, can lead to gastro-jejuno-colic fistula.
6. Gastric Remnant Complications - Bile reflux gastritis – Billroth I and II
- Gastroparesis
- B12 deficiency
- Gastric remnant malignancy – commonest at the anastomosis
- Efferent Complications
- Obstruction secondary to adhesions, local recurrence, intussusception or internal hernia.
- Hiatus hernia
- Early dumping syndrome secondary to rapid fluid shift.
- Late dumping syndrome secondary to rapid CHO absorption.
Which is most correct regarding internal hernia after bariatric laparoscopic Roux-en-Y gastric
bypass? [B3 Q26]
A. Incidence of SBO secondary to internal lesions 0.2-5.0 %
B. Occur more frequently in open than laparoscopic procedures
C. The obstruction commonly occurs in the efferent limb
D. Physical examination detects the problem with high sensitivity
E. Following Roux-en-Y gastric bypass, the stomach should be relatively distended/fluid filled
Incidence of SBO secondary to internal lesions 0.2-5.0 %
Hernia more common in laparoscopic approaches occur in the mesocolic, Peterson’s and meso-
mesenteric defects. Obstruction commonly occurs in the afferent (biliopancreatic) limb causing
distension of the very proximal jejunum, duodenum, and remnant stomach. Remnant stomach
should be relatively decompressed.
Regarding laparoscopic adjustable gastric banding: [B3 Q27]
A. Is connected by tubing to a port anterior to the rectus sheath
B. On scout films, the band should be parallel to the Gastro Oesophageal Junction (GOJ)
C. When patient ingests contrast, position of band best assessed on lateral decubitus projection
D. With band slippage, the pouch is narrowed
E. Perforation rates are typically 1-2%
Is connected by tubing to a port anterior to the rectus sheath
Band should be perpendicular to GOJ on scout film. Position is best assessed straight AP or
slightly RPO. Band slipping leads to eccentric pouch dilation. Perforation typically <0.5%
A 41-year-old woman with morbid obesity presents with a plateau in weight loss 12 weeks
after laparoscopic gastric banding. She undergoes a contrast swallow, which demonstrates
concentric dilatation of the neo-stomach with a widely patent stoma. What is the most
appropriate management? [B4 Q81]
a. no action necessary
b. nutritional advice
c. prompt decompression of the stoma by the radiologist
d. fluoroscopically guided band inflation
e. surgical replacement of the gastric band
Nutritional advice
Laparoscopic gastric banding involves laparoscopic placement of an inflatable gastric band
across the proximal stomach, forming a small fundal neo-stomach or pouch. The band is
connected to a subcutaneous port that can be accessed percutaneously to allow inflation or
deflation of the band and adjustment of stomal width and degree of hold-up. The commonest
postoperative complication is dilatation of the pouch. Three main types are described. (1) Acute
concentric pouch dilatation is due to band overinflation and is seen as a pre-stenotic dilatation proximal to an obstructed stoma. It presents as acute dysphagia and requires prompt
decompression of the stoma. (2) Chronic concentric pouch dilatation with a widely patent
stoma is seen in patients who continue to overfill their neo-stomach after surgery. Nutritional
advice is required. (3) Eccentric pouch dilatation occurs due to slippage of the band and
requires complete decompression and surgical replacement of the band. A plateau in weight
loss may also be due to loss of effect of band tightening. At fluoroscopy, fluid may be injected
to tighten the band to achieve an optimal stomal width of 3–4mm.
Small Bowel Obstruction (Includ: Large Bowel)
A patient presents to the surgical team with central abdominal pain and vomiting associated
with abdominal distension. The abdominal x-ray (AXR) reveals numerous dilated loops of
small bowel. A CT scan is carried out. Which of the following statements regarding CT
imaging in small bowel obstruction is accurate? [B1 Q20]
A. Small bowel mural hyper-density is a feature and is due to vasodilatation seen in early
ischaemia.
B. Oral contrast is mandatory for the investigation of small bowel obstruction.
C. Small bowel mural thickening is due to increased venous pressure.
D. Absence of small bowel mural enhancement is a feature of ischaemic gut secondary to
emboli rather than small bowel obstruction.
E. Lack of small bowel pneumatosis excludes ischaemia of the gut.
Small bowel mural hyper-density is a feature and is due to vasodilatation seen in early
ischaemia.
Multi-detector CT (MDCT) has been found to correlate with pathological processes in small
bowel obstruction. The earliest appearance is increased mural density due to hyperaemia.
Wall thickening is due to increasing capillary permeability, which causes submucosal oedema.
Dilatation is secondary to oedema that limits peristalsis. Lack of enhancement occurs when the
bowel dilates and compresses the capillary bed. Pneumatosis is secondary to mucosal
ischaemic change, which allows luminal air to track into the wall. Lack of enhancement is also
seen in embolic ischaemia but is not a specific sign of this process. Whilst oral contrast is
preferred in many centres, as it can help define if complete obstruction is present, it is not
mandatory. Some centres prefer the negative contrast provided by the fluid in the bowel lumen.
Patients with small bowel obstruction are also often unable to tolerate oral contrast due to
vomiting.
A 20-year-old woman with anorexia nervosa presents with intermittent abdominal pain and
vomiting relieved by lying prone. Barium meal examination reveals a vertical band-like
narrowing of the third part of the duodenum, with proximal duodenal dilatation and vigorous
to-and-fro peristalsis. What is the most likely diagnosis? [B4 Q28]
a. duodenal duplication cyst
b. annular pancreas
c. Ladd’s bands
d. superior mesenteric artery syndrome
e. duodenal atresia
Superior mesenteric artery syndrome
The third part of the duodenum is bounded posteriorly by the aorta, and anteriorly by the root
of the mesentery carrying the superior mesenteric artery. In superior mesenteric artery
syndrome, the third part of the duodenum is compressed by the superior mesenteric artery, and the angle between it and the aorta narrows to 10–22 (normal 45–65 ). The condition is
associated with severe weight loss, prolonged bedrest (particularly in a body cast), lumbar
lordosis and pregnancy. Patients report intermittent abdominal pain and vomiting, relieved by
lying prone or in the knee–elbow position. Duodenal atresia causes complete obstruction
usually distal to the ampulla of Vater and presents in neonates with a ‘double-bubble’ sign on
plain abdominal radiograph. Annular pancreas is usually asymptomatic but may present with
abdominal pain and vomiting, and barium meal demonstrates narrowing of the second part of
the duodenum. Duodenal duplication cysts cause extrinsic compression of the first and second
portions of the duodenum. Ladd’s bands are congenital peritoneal bands occurring in
association with malrotation that may cause obstruction of the second part of the duodenum,
but presentation is usually in infants and children.
A patient presents to A&E with severe upper abdominal pain 4 days following a barium enema.
There is no free air under the diaphragm on the erect CXR. There is mild elevation of the
inflammatory markers, but the surgeon is concerned with the degree of peritonism and requests
a CT scan of abdomen. On this, the small bowel is dilated to 5 cm, but is not thick walled. The
vascular structures enhance normally. There is inflammatory change noted around the
duodenum. Linear areas of low attenuation are noted extending from the porta hepatis into the
liver parenchyma. These do not extend to the margin of the liver and are in general central in
their location. The Hounsfield attenuation value of these areas is approximately –1500 HU.
Barium in the rectum obscures the images of the pelvis. What is the most likely pathology?
[B1 Q34]
A. Cholecysto-duodenal fistula.
B. Mesenteric infarction.
C. Acute bowel obstruction.
D. Perforated duodenal ulcer.
E. Complication of barium enema.
Cholecysto-duodenal fistula
The other answers are all causes of portal air, whereas the salient description is for air in the
biliary tree.
A 55-year-old man with a previous history of liver transplantation presents with a 1-week
history of abdominal pain and distension. An AXR shows some distended small bowel loops
centrally within the abdomen. You are asked to perform a CT scan of abdomen for further
evaluation. This shows a cluster of non-encapsulated dilated small bowel loops adjacent to the
anterior abdominal wall on the right side. There are adjacent crowded mesenteric vessels. What
is the most likely diagnosis?
A. Small bowel adhesions.
B. Left paraduodenal hernia.
C. Right paraduodenal hernia.
D. Foramen of Winslow hernia.
E. Trans-mesenteric hernia.
Trans-mesenteric hernia
This is when small bowel herniates through a defect in the mesentery and is compressed against
the abdominal wall, with little overlying omental fat at most levels of anatomic section through
the herniated bowel. There will be some degree of compression, crowding, displacement, and
obstruction of both the bowel and blood vessels. They are usually seen in association with
previous abdominal surgery and the creation of a Roux-en-Y anastomosis when the hernia
occurs in a surgically created defect in the mesentery.
A left-sided paraduodenal hernia is via the paraduodenal (lateral to the fourth part) mesenteric
fossa of Landzert, close to the ligament of Treitz. The characteristic features include a sac-like
mass of dilated bowel lateral to the ligament of Treitz, which displaces and indents the adjacent
stomach and transverse colon.
A right paraduodenal hernia occurs via the jejunal mesentericoparietal fossa of Waldeyer. A
cluster of dilated small bowel loops is seen lateral and inferior to the descending duodenum.
A 60-year-old female has a plain abdominal film which shows a grossly distended segment of
bowel. Which one of the following features makes a diagnosis of caecal volvulus more likely
than sigmoid volvulus? [B2 Q4]
a. Pelvic overlap sign
b. Apex lying above the level of T10
c. Liver overlap sign
d. Coffee bean sign
e. Presence of haustral markings
Presence of haustral markings
Sigmoid and caecal volvulus can sometimes be difficult to differentiate on plain abdominal
film. With caecal volvulus the haustral markings are typically present, whereas these are
usually absent in sigmoid volvulus. The pelvic overlap, liver overlap and coffee bean signs are
typical of sigmoid volvulus. In sigmoid volvulus the apex lies high in the abdomen underneath
the left hemi-diaphragm, typically above the level of T10.
Signs associated with Sigmoid Volvulus [My note]
Coffee Bean Sign
Pelvic Overlap Sign
Liver Overlap Sign
Northern Exposure Sign
Loss of Haustration
A 78-year-old man presents with abdominal pain. A plain abdominal radiograph demonstrates
a distended, inverted U-shaped loop of bowel devoid of haustra, extending from the left iliac
fossa inferiorly to just beneath the left hemidiaphragm superiorly. What is the most likely
diagnosis? [B4 Q26]
a. caecal volvulus
b. sigmoid volvulus
c. paralytic ileus
d. large bowel obstruction due to distal malignancy e. small bowel malrotation and volvulus
Sigmoid volvulus
Sigmoid volvulus usually occurs when the sigmoid loop twists around its mesenteric axis,
creating a closed loop obstruction. Typical features are of an inverted U-shaped loop
converging on the left side of the pelvis. The bowel loop is usually markedly distended, appears
ahaustral, and may overlap the lower border of the liver (liver overlap sign) or the haustrated
dilated descending colon (left flank overlap sign). The apex of the volvulus usually lies under
the left hemidiaphram with its apex above the level of T10. Caecal volvulus occurs when the
caecum is on a mesentery and involves the caecum either twisting and inverting so the caecal
pole lies in the left upper quadrant or twisting in an axial plane so that the caecum remains right
sided or central. Appearances are of a large, gas-distended viscus, usually with haustral
markings, and occasionally the gas-filled appendix may be identified.
A 27-year-old male has recurrent admissions for intermittent low-grade small bowel
obstruction of unknown cause. Which one of the following investigations would be most
appropriate? [B2 Q15]
a. Contrast-enhanced CT abdomen and pelvis
b. Barium meal
c. Small bowel enteroclysis
d. Serial abdominal plain films
e. Barium follow-through
Small bowel enteroclysis
Small bowel enteroclysis is the most appropriate examination. CT is sensitive for high-grade
obstruction as it will readily identify the level of obstruction and can demonstrate
complications such as ischaemia and perforation. Enteroclysis is the preferred investigation for
recurrent low-grade obstruction as it is more likely to demonstrate the presence of a transition
point (for example from non-obstructing adhesions) because the bowel is distended.
The examination involves passing a nasojejunal tube just distal to the duodenojejunal flexure
and distending the small bowel using either dilute barium or a double-contrast examination
with high-density barium and methylcellulose.
A 71-year-old female is admitted via A&E with abdominal pain, abdominal distension, and
vomiting. Plain abdominal film shows multiple dilated loops of small bowel. In addition, there
is gas projected over the liver shadow which is prominent centrally and has a branching
appearance. Gas is not visible over the periphery of the liver. No other abnormality is seen on
the plain film. Which of the following diagnoses is most likely? [B2 Q29]
a. Small bowel perforation
b. Small bowel infarction
c. Gallstone ileus
d. Emphysematous cholecystitis
e. Pneumatosis intestinalis
Gallstone ileus
Specific signs of gallstone ileus can be seen on the plain abdominal film in up to 40% of
patients. Fifty per cent of patients have evidence of small bowel obstruction and up to 30%
have gas in the biliary tree. Biliary tree gas is typically more prominent centrally and spares
the periphery of the liver, whereas portal venous gas is more easily visualised in the periphery
of the liver, which may be associated with small bowel infarction. The gallstone most
frequently lodges in the terminal ileum but is often not seen on the plain film. The presence of
small bowel obstruction, pneumobilia and a visible stone are called Rigler’s triad.
13 months post-Roux-en-Y bariatric gastric bypass surgery a 45-year-old woman presents with
abdominal pain, vomiting and constipation. Abdominal X-ray shows Small Bowel Obstruction
(SBO) on CT. Which is the most likely cause of the SBO if the site of the obstruction is in the
alimentary limb? [B3 Q12]
A. Internal hernia
B. Jejeno-jejunostomy stricture
C. Intussusception of the Roux limb
D. Intraluminal haematoma
E. Meso-colic haematoma
B-D occur < 12 months, whereas internal hernias and meso-colic constriction of the Roux limb
are more chronic complications. Chronic complications with obstruction at the biliopancreatic
limb, internal hernias and anastomotic strictures at the common channel are internal hernias
and adhesions.
A 68-year-old woman presents with small bowel obstruction and undergoes contrast-enhanced
CT of the abdomen. This demonstrates dilated small bowel to the level of the mid-ileum, where
a herniated loop of small bowel is seen emerging inferolateral to the left pubic tubercle. What
is the most likely cause of small bowel obstruction in this patient? [B4 Q8]
a. femoral hernia
b. indirect inguinal hernia
c. direct inguinal hernia
d. spigelian hernia
e. obturator hernia
Femoral hernia
External hernias are the second most common cause of small bowel obstruction after adhesions.
A femoral hernia protrudes through the femoral ring, lying medial to the femoral vein, which
may be compressed. On CT the hernia is seen inferolateral to the pubic tubercle, in contrast to
inguinal hernias, which usually lie superomedial to the tubercle, though differentiation may be
difficult in non-incarcerated cases. Femoral hernias are more prone to incarceration due to the
inflexible margins of the femoral ring. Inguinal hernias may be classified as indirect (passing
down the inguinal canal, seen lateral to the inferior epigastric vessels) or direct (protruding
directly through the lower abdominal wall medial to the inferior epigastric vessels). A spigelian
hernia protrudes through a defect in the inferolateral anterior abdominal wall. Obturator hernias
protrude through the obturator foramen, between the pectineus and external obturator muscles.
A 68-year-old woman presents with abdominal pain, distension, and vomiting. Plain abdominal
radiograph demonstrates bowel obstruction, gas within the biliary tree, and an ectopic, calcified,
3 cm gallstone. What is the most likely site of bowel obstruction? [B4 Q70]
a. pylorus
b. duodenum
c. proximal ileum
d. terminal ileum
e. sigmoid
Terminal ileum
Gallstone ileus accounts for up to 5% of intestinal obstruction, increasing in prevalence with
age. It involves erosion of a large gallstone from the gallbladder or common bile duct into the
bowel, which goes on to cause obstruction. The classic appearance on plain film (Rigler’s triad)
is only seen in 10% of cases and consists of partial or complete intestinal obstruction (usually
small bowel), gas in the biliary tree and an ectopic calcified gallstone. The most common site
of fistulous communication is between the gallbladder and the duodenum, seen in 60%, and
this may be demonstrated on barium meal as a contrast collection lateral to the first part of the
duodenum representing barium within the gallbladder. Fistulas occur less commonly between
the common bile duct and duodenum, gallbladder and colon. The ectopic gallstone most often
causes obstruction at the terminal ileum (60–70%), followed by the proximal ileum, distal
ileum, pylorus, sigmoid and duodenum.
A 35-year-old male patient from the Indian subcontinent presents with a 2-month history of
lower abdominal pain, per rectum (PR) bleeding, and weight loss. His haemoglobin is 9.4 and
C-reactive protein (CRP) is 123. The patient is tender in the RIF. A CT scan is performed due
the suspicion of appendiceal pathology, but unusual history. This shows bowel wall thickening
of the terminal ileum with mild proximal bowel dilatation. The inner bowel wall is hypodense
with enhancement of the outer bowel wall. There is stranding in the fat, which causes mass
effect displacing other loops of bowel. Mild regional adenopathy is noted. The appendix is not
visualized, but the caecum appears normal. There is a similar area of bowel wall thickening in
the sigmoid colon. What is the most likely diagnosis? [B1 Q22]
A. Yersinia.
B. Tuberculosis.
C. Lymphoma.
D. Crohn’s disease.
E. Carcinoid
Crohn’s disease
The findings described are classical for Crohn’s disease and lymphadenopathy is seen in up to
30% of cases. Tuberculosis more typically involves the caecum. Lymphoma usually causes a
nodular appearance to the bowel. It is not associated with stricturing of the affected segment
and is more classically associated with dilatation of the affected segment due to destruction of
the myenteric plexus.
A 40-year-old male with a 22-year history of Crohn’s disease presents with abdominal pain,
diarrhoea, and low-grade fever. To attempt to limit his lifetime radiation exposure he is
investigated via MR enterography. Which of the following MRI findings is the earliest in active
inflammation? [B1 Q73]
A. Increased mesenteric vascularity.
B. Small bowel wall thickening.
C. Mucosal hyperenhancement.
D. Peri-enteric inflammation.
E. Reactive adenopathy
Mucosal hyperenhancement
The lack of ionizing radiation is a major advantage to MRI for patients with Crohn’s disease
given the chronic nature of this condition necessitating frequent investigation. MRI can be
performed via enterography or enteroclysis. In enterography, large volumes of fluid (or a fluid
inducing laxative) are ingested. Enteroclysis involves administration of enteric contrast
material via a naso-enteric tube. Sequence acquisition involves fat suppression and intravenous
contrast. Increased mucosal hyperenhancement (compared with that seen in normal
surrounding loops) may be the earliest sign of active inflammation, even in the absence of wall
thickening. Increased vascularity, peri-enteric inflammation, and reactive adenopathy are other
signs of active Crohn’s disease. In severe Crohn’s disease mucosal hyperenhancement
combined with submucosal oedema gives a ‘stratified’ appearance. Serosal hyperenhancement may also be seen, giving a ‘target’ appearance. Mural thickening is defined as greater than 3
mm, although an under-distended bowel may mimic this finding.
Which of the following most favours Crohn’s versus pseudomembranous colitis? [B3 Q6]
A. Ascites
B. Absence of small bowel involvement
C. Fibro fatty mesenteric proliferation around involved colon
D. Colonic wall thickening of 11mm
E. Low attenuation mural thickening (accordion sign)
Fibro fatty mesenteric proliferation around involved colon is more likely to indicate Crohn’s
disease.
A small bowel series is requested for a patient who has a history of systemic sclerosis. Which
of the following is a feature of small bowel systemic sclerosis? [B1 Q8]
A. Stacked coin appearance due to infiltration of small bowel loops.
B. Pseudo-diverticula affecting the anti-mesenteric side of the bowel.
C. Decreased intestinal transit time.
D. Small bowel systemic sclerosis is only seen in 10% of patients with systemic sclerosis, but
the disease is rapidly progressive when it is present.
E. Pneumatosis intestinalis.
Pneumatosis intestinalis
The stacked coin appearance is seen secondary to intramural haemorrhage—the appearances
of systemic sclerosis are of tightly packed folds of normal thickness in a dilated portion of
bowel, which has been given the title ‘accordion’ or ‘hidebound’ bowel.
The pseudo-diverticula (10–40%) are seen on the mesenteric side of the bowel, unlike colonic
diverticula.
The transit time is prolonged, as there is reduced intestinal motility. Another classical feature
is of a markedly dilated duodenum, due to the loss of the enteric innervations—mega
duodenum. This classically terminates abruptly at the level of the superior mesenteric artery
(SMA).
Pneumatosis cystoides can occur in systemic sclerosis of the small bowel. Small bowel disease
is seen in up to 40% of patients with systemic sclerosis and indicates rapidly progressing
disease.
Scleroderma [Core Radiology]
- Def: a systemic disease characterized by the deposition of collagen into multiple internal
organs and the skin. - Replacement of the muscular layers with collagen, resulting in
- Slowed transit
- Bacterial overgrowth
- Progressive dilatation
- Radiographically
- Sacculation on the antimesenteric border
- Hidebound Bowel – thin, straight bowel fold stacked together.
Intestinal Scleroderma [STATdx]
- Progressive systemic sclerosis
- GI tract: Most common internal organ system involvement (80-90%)
o Oesophagus > duodenum > anorectal > small bowel > colon - Oesophagus
o Atony or aperistalsis (lower 2/3 smooth muscle)
o Mild to moderate dilatation
o Patulous lower oesophageal sphincter (early)
o Ulcers, fusiform peptic stricture (late)
o GORD, Barrett’s oesophagus - Small bowel
o Marked dilatation of small bowel, especially duodenum and jejunum
o Duodenal findings identical to superior mesenteric artery (SMA) syndrome
o Hidebound small bowel: Atonic with closely spaced, thin folds, sacculation
(pathognomonic of scleroderma)
o Prolonged transit time with barium retention in duodenum and small bowel up
to 24 hours
o Penumatosis intestinalis and pneumoperitoneum
o Transient, nonobstructive intussusceptions - Colon
o Sacculation on border of transverse and descending colon
o Loss of haustrations
o Stercoral ulceration (from retained faecal material in rectosigmoid) - Top Differential Diagnoses
o SMA syndrome
o Celiac-sprue disease
o Ileus
A 71-year-old female with scleroderma undergoes a barium swallow examination. Which one
of the following findings concerning the oesophagus would not be consistent with this
diagnosis? [B2 Q1]
a. Oesophageal dilatation
b. Superficial ulcers
c. Hypoperistalsis in the upper third of the oesophagus
d. Stricture 5cm above the gastro-oesophageal junction
e. Oesophageal shortening
Hypoperistalsis in the upper third of the oesophagus
The oesophagus is the most involved location of the gastro-intestinal tract in patients with
scleroderma. Smooth muscle atrophy causes hypoperistalsis and eventually aperistalsis in the
lower two-thirds of the oesophagus. The upper third of the oesophageal wall contains skeletal
muscle and is therefore unaffected by the disease process.
A 41-year-old female with a background of arthralgia, chronic abdominal pain, and diarrhoea
is investigated via a small bowel series. Findings include a prolonged transit time, and dilated
loops of small bowel with normal appearing valvulae and pseudodiverticula. What is the most
likely diagnosis? [B1 Q53]
A. GI scleroderma.
B. Behcet’s disease.
C. Whipple disease.
D. Small bowel lymphoma. E. Coeliac disease.
GI scleroderma
Deeply penetrating ulcers are seen in Behcet’s disease. Whipple disease is an extremely rare
form of intestinal lipodystrophy. Thickening of jejuna folds is seen, but there is little or no
small bowel dilatation and small bowel transit time is normal. Pseudodiverticula are not seen
in coeliac disease. The valvulae are thickened in lymphoma.
Which is the most affected site in systemic sclerosis after the oesophagus? [B3 Q16]
A. Anorectal
B. Small bowel
C. Colon
D. Stomach
E. Oropharynx
Oesophagus is most frequently affected (75-90%) followed by anorectum (50-70%), small
bowel (40%) and colon (10-50%). The stomach is the least affected in the GIT
Which of the following is a case of decreased/absent duodenal folds? [B3 Q38]
A. Scleroderma
B. Pancreatitis
C. Lymphoma
D. Melanoma metastases
E. Whipple’s disease
Scleroderma
Scleroderma, strongyloides, and cystic fibrosis are causes of decreased/absent folds. Crohn’s
and amyloidosis can cause decreased or increased folds.
A 44-year-old man presents with a vague history of central abdominal pain and mild weight
loss. On further questioning, there are other features in the history suggestive of malabsorption.
Amongst other investigations, a CT scan of abdomen is requested. This shows dilated fluid-
filled small bowel loops and multiple enlarged mesenteric lymph nodes, encasing the
mesenteric vessels. The lymph nodes are of homogeneous soft tissue density. What is the most
likely cause of the CT findings? [B1 Q59]
A. Whipple’s disease.
B. Coeliac disease complicated by lymphoma.
C. Cavitating mesenteric lymph node syndrome.
D. Abdominal tuberculosis.
E. Castleman disease.
Coeliac disease complicated by lymphoma
Whipples disease, cavitating mesenteric lymph node syndrome, and abdominal TB more
typically have mesenteric lymph node enlargement that has central low attenuation, rather than
being of homogeneous soft-tissue density. Whipple disease is a systemic bacterial infection
caused by Tropheryma whippelii. Lymph nodes affected by Whipple disease typically have a
high fat content, causing the low attenuation, usually between 10 and 20 HU.
Cavitating mesenteric lymph node syndrome is associated with coeliac disease. The lymph
nodes are truly cavitating and usually regress following a gluten-free diet.
The lymph nodes in abdominal tuberculosis typically have caseous necrosis and thus central
low density on CT.
Castleman disease causes benign masses of lymphoid tissue of unknown aetiology. It can cause
mesenteric lymphadenopathy, which is homogeneous, but the disease itself is rare and
mesenteric involvement is much less common than mediastinal involvement.
A 45-year-old man has a long history of intermittent diarrhoea, abdominal bloating, and cramps,
but has neglected to seek medical advice until now. His GP is worried about undiagnosed
Crohn’s disease and sends him for a small bowel series. This shows some dilatation of the
proximal small bowel, with segmentation and flocculation of the barium and an increased
number of normal thickness folds seen in the ileum. There is no evidence of stricture formation
or ulceration. What is the most likely underlying diagnosis? [B1 Q72]
A. Amyloidosis.
B. Chronic ischaemic enteritis.
C. Whipple’s disease.
D. Coeliac disease.
E. Lymphoma.
Coeliac disease
The segmentation and flocculation of barium are findings on a small bowel series that are
typical of malabsorption and therefore the most likely diagnosis is coeliac disease. Other findings in coeliac disease include dilatation, a granular appearance to the barium secondary to
hypersecretion, jejunization of the ileum, and the ‘moulage’ sign. The latter refers to a smooth,
tubular appearance to the jejunum in longstanding coeliac disease, secondary to atrophy and
effacement of the jejunal mucosal folds.
Lymphoma can be a complication of coeliac disease and generally causes shallow, ulcerated
masses or the development of thickened, nodular small bowel folds.
Whipple’s disease, amyloidosis, and chronic ischaemic enteritis all cause thickening of the
small bowel folds.
A 31-year-old male is investigated as an outpatient for diarrhoea. A small bowel meal study
reveals jejunal dilatation with thickened valvulae conniventes. In the ileum an increased
number of mucosal folds are seen. Which of the following diagnoses is most likely? [B2 Q27]
a. Lymphoma
b. Crohn’s disease
c. Coeliac disease
d. Whipple disease
e. Behcet syndrome
Coeliac disease
Jejunal dilatation and jejunisation of the ileal loops are characteristic features of coeliac disease.
This is an immunological intolerance to gluten that causes villous atrophy in the small intestine.
In Whipple disease there is thickening of the jejunal and duodenal mucosal folds but typically
no luminal dilatation. Dilatation of the small bowel does occur with lymphoma but jejunisation
of the ileum is not a feature.
A 34-year-old female is investigated for intermittent abdominal pain and malabsorption. Small
bowel meal shows dilatation of the proximal small bowel loops but a normal mucosal fold
pattern. Which one of the following is the most likely underlying diagnosis? [B2 Q35]
a. Coeliac disease
b. Amyloid
c. Whipple disease
d. Giardiasis
e. Eosinophilic gastroenteritis
Coeliac disease
All of these may cause malabsorption. Amyloid can cause dilatation but also causes diffuse
thickening of the valvulae conniventes throughout the small bowel. With Whipple disease and
eosinophilic gastroenteritis, one would not see dilatation of the bowel, but thickening of the
mucosa is again a prominent feature. Giardiasis causes thickening and marked distortion of the
mucosal folds in the duodenum and jejunum. One of the hallmark features of untreated coeliac
disease is jejunal dilatation. Typically, the mucosal folds are of normal thickness.
A 34-year-old woman with a history of steatorrhoea and weight loss undergoes a small bowel
follow-through examination that demonstrates dilatation of the proximal small bowel with flocculation and segmentation of the barium column. Fold thickness is normal. What is the
most likely diagnosis? [B4 Q5]
a. Crohn’s disease
b. Zollinger–Ellison syndrome
c. coeliac disease
d. small bowel lymphoma
e. Whipple’s disease
Coeliac disease
Coeliac disease is characterized by malabsorption due to intolerance to the alpha-gliadin
component of gluten, which causes small intestinal villous atrophy. Typical findings are of
dilatation of the proximal small bowel, together with dilution of the barium column due to
hypersecretion of fluid. Artefacts such as segmentation (breaking up of the barium column) or
flocculation (clumping of disintegrated barium) were traditionally classic features of coeliac
disease but are less often seen nowadays with improved barium suspensions. In Whipple’s
disease the small bowel is typically non-dilated and shows moderate fold thickening. Crohn’s
disease usually causes nodular fold thickening, and predominantly involves the distal small
bowel. Zollinger–Ellison syndrome results in dilatation of proximal small bowel due to
hypersecretion, but typically causes thickened folds. Small bowel lymphoma is usually
associated with fold thickening.
A 60-year-old woman presents with weight loss and diarrhoea. CT of the abdomen
demonstrates multiple, enlarged, low-attenuation mesenteric lymph nodes containing fat–fluid
levels and splenic atrophy. What is the most likely diagnosis? [B4 Q55]
a. tuberculosis
b. coeliac disease
c. Whipple’s disease
d. lymphoma
e. metastatic squamous cell carcinoma
Coeliac disease
Cavitating mesenteric lymph node syndrome is a rare complication of coeliac disease, in which
multiple enlarged lymph nodes are seen in the jejunoileal mesentery. The nodes have central
low attenuation and may contain fat or fluid, or fat–fluid levels. Splenic atrophy is usually seen,
and jejunal or duodenal biopsy confirms villous atrophy of the small bowel mucosa. Low-
attenuation lymphadenopathy may also be seen in tuberculosis, Whipple’s disease, lymphoma,
and necrotic metastases, but fat–fluid levels have been reported only in coeliac disease
A 68-year-old male patient has a 20-year history of RA. During a recent flare he was
commenced on steroid therapy, although this has now been discontinued. The patient is now
complaining of mild abdominal discomfort, diarrhoea, and mild weight loss. A barium meal is
performed, but is suboptimal, as the patient is poorly mobile. Within the limitations of the study,
there is reduced peristalsis in the oesophagus and mild reflux. The antrum of the stomach is
felt to be mildly narrowed and rigid. Thickened rugal folds are noted. A subsequent small bowel
series is carried out. The jejunal folds measure 4 mm and the ileal folds appear more plentiful,
and measure 3 mm. Contrast is present in the caecum at 4 hours. Spot screening of the terminal
ileum reveals the same findings as those described above. What is the most likely diagnosis?
[B1 Q16]
A. Gastric erosions.
B. Whipple’s disease.
C. Mastocytosis.
D. Amyloidosis.
E. Crohn’s disease.
Amyloidosis.
This patient probably has amyloidosis secondary to prolonged RA. GI involvement is more
common in primary (70%) than secondary (13%) amyloidosis. Nevertheless, the small bowel
is involved in 74% of cases of GI amyloidosis and secondary amyloidosis is the most common
type of amyloid disease. Amyloidosis is secondary to the deposition of insoluble amyloid
protein in soft tissues and organs. In primary amyloidosis the heart (90%), followed by the
small bowel and the lungs (70%), are the most affected organs. The kidneys are affected in 90%
of cases of secondary amyloidosis. Amyloidosis classically causes a diffuse thickening of
bowel folds. It may cause dilated bowel folds if the myenteric plexus is involved. The main
differential for amyloid is Whipple’s disease and intestinal lymphangiectasia. Whipple’s
disease does not cause bowel dilatation or rigidity, as described in the antrum in this patient.
Crohn’s disease can also present with thickened folds, but it is more commonly focal with the
most pronounced abnormality in the terminal ileum. Ulceration is also commonly seen in
Crohn’s, but 68 years old would be a late first presentation for Crohn’s. Whilst option A is true,
this is not what the question asked. Patients with mastocytosis most commonly present in
infancy .
GI Amyloidosis [STATdx]
Fluoroscopic findings:
- Stomach
o Diffuse or focal thick folds (nodular or mass-like) with calcification - Small intestine
o Symmetrical regular thickening of folds (with or without nodular component)
o Impaired motility often with small bowel dilatation, and
o Slow transit - Colon
o Luminal narrowing
o Loss of haustrations, and o Thickened transverse folds
CT findings:
- Small bowel, large bowel, and stomach
o Wall thickening – focal/nodular mass-like or diffuse, calcified, may infiltrate
into mesentery
o distention (due to hypomotility)
o intramural haemorrhage,
o luminal narrowing (due to wall thickening), and
o intussusceptions - Liver and spleen
o Hepatomegaly or splenomegaly may be only manifestation
o Focal, geographic, or diffuse hypoattenuation in liver (mimic steatosis)
o Frequently in periportal distribution (with parenchymal calcification) - Spleen
o Splenomegaly
o Focal or diffuse hypoattenuation (with parenchymal calcification)
o Increased risk of spontaneous rupture - Gallbladder wall thickening (extremely rare)
- Retroperitoneal soft tissue infiltration resembling retroperitoneal fibrosis
- Mesenteric infiltration resembling sclerosing mesenteritis
- Lymphadenopathy with calcification
- Mass-like amyloidomas in soft tissues with calcification
MRI findings:
Involved site – Low T2, intermediate to high T1
Whipple Disease [Core Radiology]
Def: – A rare systemic disease caused by Tropheryma whipplei, multiple systemic
manifestations, including chronic diarrhoea and malabsorption
Imaging:
- Fluoroscopic small bowel follow-through or enteroclysis
* Common site – Distal duodenum and jejunum
* Severe cases – Distal Ileum
* Thickened, irregular folds with sand-like micronodules
* Normal or mildly dilated lumen
* Thickened mesentery and separation of bowel loops. - CT
* Mesenteric and retroperitoneal lymphadenopathy with near fat density. (Digested bacilli)
* Thickened proximal small bowel folds with or without submucosal oedema due to
hypoalbuminemia * Ascites, splenomegaly, pneumatosis intestinalis. - MR
Lymph nodes may show increased T1 signal due to fat
Top Differential Diagnoses:
- Celiac disease
- Intestinal opportunistic infections
- Dysgammaglobulinemia
- Intestinal metastases and lymphoma
Clinical Issues:
- Mostly affects middle-aged White men
- Prodromal phase: Fevers, fatigue, arthralgias, arthritis
- Late phase: Diarrhea, malabsorption, steatorrhea, adenopathy, abdominal pain
- GI symptoms generally later manifestation of disease
- Can be fatal without therapy (long-term antibiotics)
- Clinical symptoms often subside quickly after therapy
A 45-year-old man, with a history of AIDS, has a 3-month history of abdominal pain and
weight loss. A CT scan of abdomen is performed which shows ascites with peritoneal
thickening, several areas of mural thickening in the small bowel, and multiple low attenuation
lymph nodes. Which one of the following infections is most likely? [B1 Q59]
A. CMV infection.
B. TB.
C. Cryptosporidiosis.
D. Amoebiasis.
E. Campylobacter.
TB.
Cryptosporidiosis is the most common cause of enteritis in AIDS patients. It more commonly
causes proximal small bowel thickening in the duodenum and jejunum, and CT may show
small lymph nodes.
CMV infection of the small bowel can show a terminal ileitis indistinguishable from Crohn’s
disease.
The typical CT findings in amoebiasis are thickening of the right colonic wall and a rounded
abscess in the right lobe of liver with a peripheral zone of oedema.
TB usually shows ileocaecal involvement, low attenuation mesenteric nodes, and ascites with
peritoneal thickening. Mycobacterium avium intracellulare may also occur with low
attenuation mesenteric nodes and thickening of small bowel folds.
What is the most common site of involvement in tuberculosis of the gastrointestinal tract? [B4
Q83]
a. stomach
b. duodenum
c. ileo-caecal region
d. splenic flexure
e. rectum
Ileo-caecal region
Tuberculosis of the gastrointestinal tract may occur through ingestion of infected sputum, or
by haematogenous spread to submucosal lymph nodes from a pulmonary tuberculous focus. It
most commonly affects the ileo-caecal region due to its abundance of lymphoid tissue and
relative stasis of gut contents. Typical features at this site include circumferential thickening
of the terminal ileum and caecum, a thickened ileo-caecal valve and ulceration following the
orientation of lymphoid follicles (longitudinal in the terminal ileum and transverse in the colon).
Marked enlargement of adjacent mesenteric lymph nodes with central areas of low attenuation
may be seen
A 38-year-old patient with AIDS presents with diarrhoea and steatorrhoea. As part of the work-
up, small bowel enteroclysis shows thickened jejunal folds with nodularity and evidence of marked jejunal spasm. The ileum has normal appearances. Which one of the following is the
most likely underlying cause? [B2 Q32]
a. Cytomegalovirus
b. Tuberculosis
c. Mycobacterium avium intracellulare
d. Cryptosporidium
e. Giardiasis
Giardiasis
All the stems are potential causes for these symptoms in a patient with AIDS, however
giardiasis is the most likely cause given these imaging appearances. Cytomegalovirus most
typically affects the caecum, and tuberculosis affects the caecum and ileocaecal valve.
Mycobacterium avium intracellulare can affect the ileum and jejunum but does not usually
cause spasm. Cryptosporidium affects the duodenum, and the jejunum can be affected, but
dilatation is more common than spasm.
A 55-year-old woman is admitted to hospital after several episodes of melaena. She has an
upper GI endoscopy performed, which is normal. A CT scan of abdomen is requested, and this
demonstrates a large exophytic mass arising from the jejunum in the left upper quadrant. It is
heterogeneous in density and has some peripheral enhancement and central necrosis. There is
no calcification, intestinal obstruction, or evidence of aneurysmal dilatation of the affected
segment of jejunum. There is no adjacent lymphadenopathy or ascites. What is the most likely
diagnosis? [B1 Q66]
A. Adenocarcinoma.
B. Lymphoma.
C. Carcinoid tumour.
D. Metastasis.
E. Gastrointestinal stromal tumour (GIST).
Gastrointestinal stromal tumour (GIST)
At contrast-enhanced CT, GISTs appear as large exophytic masses with peripheral
enhancement. They usually have an attenuation like that of muscle, but they may have
heterogeneous attenuation, depending on their level of aggressiveness. More aggressive GISTs
may also contain a central area of necrosis.
Adenocarcinoma of the jejunum is rare, more commonly occurring in the duodenum. They also
tend to be stricturing lesions, rather than exophytic masses and may present with obstruction.
Carcinoid is also rare in the proximal small bowel, the distal ileum being a more usual location.
The primary lesion is often quite small, with the nodal metastatic lesion in the small bowel
mesentery being more conspicuous on CT. This is often spiculated (surrounding desmoplastic
reaction) and may contain calcification.
Lymphoma can have several manifestations in the small bowel, from nodular thickening of the
mucosal folds to large masses with aneurysmal dilatation of the small bowel in the affected
segment. Associated lymphadenopathy is typical.
A 57-year-old diet-compliant male patient with coeliac disease has a CT abdomen and pelvis
for the investigation of cachexia and two stone weight loss over six months. A 7 cm segment
of ileum shows mild dilatation and circumferential thickening, with multiple low-attenuation
mesenteric and para-aortic lymph nodes. Which one of the following is the most likely
diagnosis? [B2 Q59]
a. Tuberculosis
b. Gastro-intestinal lymphoma
c. Coeliac disease
d. Whipple disease
e. Crohn’s disease
**Gastro-intestinal lymphoma **
Hypoattenuating lymph nodes can be attributed to many causes, but lymphoma and
tuberculosis are the most common. Lymphoma of the gastro-intestinal tract most commonly
affects the ileum, although lymphoma associated with coeliac disease most commonly affects
the jejunum. Although 90% of tuberculosis of the gastro-intestinal tract occurs in the ileum,
lymphoma is most likely in this scenario. Dilatation of the small bowel with lymphoma is
common but obstruction is rare due to the soft pliable nature of the tumour.
A 25-year-old male presents following blunt abdominal trauma following a motor vehicle
accident. Which is the most common CT finding in the ‘shock bowel’? [B3 Q8]
A. Increased small bowel mucosal enhancement
B. Small bowel luminal dilation
C. Fluid-filled loops of small bowel
D. Colonic involvement greater than small bowel
E. Focal involvement of the small bowel
Increased small bowel mucosal enhancement (HU > psoas) and mural thickening > 3mm are
the most common signs. Colon involvement is infrequent, and the small bowel is typically
diffusely involved.
A 73-year-old woman is referred from surgical outpatients for a barium enema. She has a 3-
month history of weight loss and a microcytic anaemia. The procedure is unremarkable, and
you leave the screening room to go and continue some plain film reporting. Ten minutes later
you are contacted by one of the radiographers who was helping during the enema. She is
distressed and tells you that she found the patient collapsed in the bathroom having what
appeared to be a seizure. You immediately attend and assess the patient. She is drowsy, but
heart rate, blood pressure, and SaO 2 are normal. What is the most likely complication to have
caused her acute illness? [B1 Q43]
A. Cardiac arrhythmia secondary to rectal distension.
B. Venous intravasation.
C. Water intoxication.
D. Intramural barium.
E. Side-effect of hyoscine butyl bromide (Buscopan).
Water intoxication.
All five options are complications of barium enema. Additional potential complications include
bowel perforation, barium impaction, and transient bacteraemia. Complications during barium
enema are rare. Perforation of the bowel is the most frequent serious complication, occurring
in approximately 0.02–0.04% of patients. Venous intravasation may result in a barium
pulmonary embolus, which carries an 80% mortality. Water intoxication causes drowsiness
and convulsions, as in this case. There is an increased risk in megacolon because of the large
area of bowel mucosa available for the absorption of water. Water intoxication has also been
attributed to the preparatory laxatives used. Buscopan may cause cardiac arrhythmia and
should be used with caution in those with cardiac disease; other relative contraindications
include angle-closure glaucoma, myasthenia gravis, paralytic ileus, pyloric stenosis, and
prostatic enlargement.
About the use of glucagon in barium enema examinations, which of the following statements
is correct? [B1 Q60]
A. 0.1mg of glucagon is an appropriate dose.
B. Diabetes is a contraindication to the use of glucagon.
C. Insulinoma is a contraindication to the use of glucagon.
D. Glucagon can be safely used in patients with phaeochromocytoma.
E. Smooth muscle relaxation is optimal at 5 minutes and lasts approximately 1 hour.
Insulinoma is a contraindication to the use of glucagon
Glucagon is a potent hypotonic agent. If 1mg of glucagon is injected intravenously it takes
approximately 1 minute to work and lasts about 10–20 minutes. Intravenously administered
glucagon decreases discomfort during barium enema examinations. Glucagon administration
is generally safe but is contraindicated in patients with insulinoma and phaeochromocytoma.
Diabetes is not a recognized contraindication.
A 72-year-old man attends for a barium enema examination. He has no known allergies. In
considering administration of intravenous hyoscine-N-butylbromide (Buscopan), which factor
in his medical history is it most important to be aware of? [B4 Q35]
a. prostatism
b. type I diabetes
c. glaucoma
d. migraine
e. unstable cardiac disease
Unstable cardiac disease
Buscopan is commonly used in radiological practice as a smooth muscle relaxant. As a non-
selective muscarinic antagonist, it produces other autonomic responses including pupillary
dilatation and tachycardia and may potentially precipitate an attack of acute angle-closure
glaucoma that requires prompt treatment to prevent permanent visual loss. However, most
glaucoma is of the open-angle form, which is unaffected by Buscopan, and it is therefore
advised that routine enquiry about a history of glaucoma is unnecessary. Instead, patient
information leaflets should advise all patients to attend hospital immediately should they
develop painful, blurred vision. Routine enquiry about prostatism, porphyria and myasthenia
gravis are also not recommended. However, in patients with unstable cardiac disease, the
tachycardia and slight increase in diastolic blood pressure caused by Buscopan carry the
potential risk of arrhythmia. The presence of unstable cardiac disease is therefore deemed to
be the only potential reason to withhold Buscopan.
A 75-year-old man is undergoing a CT colonography examination for investigation of a change
in bowel habit. He has difficulty retaining the CO2 for adequate bowel distension. Which of
the following segments of colon is likely to be better distended on the prone scan? [B1 Q69]
A. Caecum.
B. Transverse colon.
C. Rectosigmoid.
D. Ascending colon.
E. Hepatic flexure.
Rectosigmoid.
On a prone scan during a CT colonography examination, the rectosigmoid is generally better
distended than on the supine scan because it is a more posteriorly placed structure and air gets
displaced to the non-dependent position. The other segments named are usually better
distended on the supine scan, particularly the caecum and transverse colon, as these are more
anteriorly placed within the abdomen
Which is the most common source of false positives in cathartically prepared CT colonography
by Computer aided detection (CAD)? [B3 Q33]
A. Haustral folds
B. Untagged/poorly tagged stool
C. Ileocecal valve D. Electronic cleansing and tagging artefact
E. Extrinsic compression
Haustral folds
Most common source of FP in CAD in cathartically prepared CT colonography.
Untagged/poorly tagged stool is the second most common FP in cathartically prepared and
most common source of FP in non-cathartically prepared CT colonography
A 37-year-old man on the intensive care unit, 3 weeks post-operatively for complicated small
bowel resection for Crohn’s disease, is noted to have an oozing from a site on his anterior
abdominal wall. His renal function remains normal and inflammatory markers are persistently
elevated with White Cell Count (WCC) 15.9 and C-reactive Protein (CRP) 124. On a CT of
the abdomen 5 days ago, no collection was demonstrated. CT fistulography is being considered.
Which is the best answer with regards to CT fistulography? [B3 Q42]
A. Conventional fistulography has a higher spatial resolution
B. Iodinated contrast should be used, diluted 1 in 100
C. Oral contrast is helpful in most cases
D. Intravenous (IV) contrast should be avoided
E. CT fistulography has a greater temporal resolution
Conventional fistulography has a higher spatial resolution
CTF helps to delineate fistulous tracks in critically ill patients where prolonged contrast studies
are not feasible. Iodinated contrast (300mg iodine/ml) should be diluted 1 in 10, with a volume
based on an estimate of the length of bowel or volume of cavity being investigated. Oral
contrast can confuse the origin of intraluminal contrast and is best avoided, whereas IV contrast
is helpful. Conventional fistulography has a higher spatial and temporal resolution.
A 54-year-old woman with a sensation of incomplete evacuation on defecation undergoes
conventional defecography. Following introduction of barium paste into the rectum, in which
position should the patient be placed for imaging? [B4 Q7]
a. supine
b. prone
c. left lateral
d. right lateral
e. sitting
Conventional defecography is used in the imaging evaluation of obstructed defecation. Barium
paste is instilled into the rectum with a Foley catheter, with the patient in the left lateral position
Prior opacification of the small bowel, bladder, vagina or peritoneum may also be performed
to aid diagnosis. With the patient sitting on a commode placed on the footrest of a standard
fluoroscopic table, static images are first obtained at rest and with contraction of the pelvic floor
muscles. A cine-loop of evacuation is then obtained until the rectum is empty or three 30-
second attempts at evacuation have been made. A variety of conditions may be demonstrated,
including rectocele, enterocele, rectal intussusception and anismus.
During double-contrast barium enema, a prone overcouch film with the tube angled 45’
caudally, centred 5 cm above the posterior superior iliac spines, is performed to optimally
visualize which segment of the large bowel? [B4 Q92]
a. caecum
b. hepatic flexure
c. transverse colon
d. splenic flexure
e. sigmoid colon
Sigmoid colon
The prone-angled, over-couch view performed as described separates overlying loops of
sigmoid colon
A 73-year-old woman presents with intermittent lower gastrointestinal bleeding and iron
deficiency anaemia. She is clinically suspected to have angiodysplasia. What are the most
likely findings on barium enema? [B4 Q73]
a. normal appearances
b. multiple small polyps in the colon
c. multiple shallow ulcers in the colon
d. multiple, serpiginous, filling defects in the colon
e. a focal, irregular, circumferential narrowing in the colon
Normal appearance
In angiodysplasia, there is degenerative dilatation of the normal vessels in the submucosa of
the bowel wall. It is associated with increasing age, and in about 20% of cases with aortic
stenosis. It occurs most commonly in the right colon and presents with intermittent, low-grade
bleeding. Barium enema shows no abnormality, as the lesion is submucosal, but increased
tracer accumulation may be seen at the site of haemorrhage on 99m Tc-labelled red cell
scanning. Angiography, if performed, may demonstrate a cluster of vessels along the
antimesenteric border during the arterial phase and early opacification of the draining ileocolic
vein.
In the staging of rectal cancer by MRI, which sequence provides optimum visualization of the
tumour? [B4 Q80]
a. T1W
b. contrast enhanced T1W
c. T2W
d. FLAIR
e. proton density
T2W
MR is a highly accurate method of local staging of rectal cancer, with better assessment of
locoregional nodal involvement than CT and clear depiction of the mesorectal fascia, allowing
accurate prediction of whether the circumferential resection margin will be tumour free. T2W images provide optimal visualization of the tumour, which appears as an intermediate signal-
intensity mass. Contrast-enhanced T1W images result in enhancement of the normal bowel
wall as well as the tumour, which may lead to upstaging. FLAIR sequences are not routinely
used for rectal cancer staging.
Ischaemic Colitis
A 78-year-old previously well female is admitted with acute abdominal pain and diarrhoea.
Contrast-enhanced CT of the abdomen and pelvis shows thickening of a 13cm segment of
proximal descending colon and mucosal hyperenhancement. The rest of the colon is normal,
and the small bowel is unaffected. There is a small amount of free fluid in the pelvis. Which
one of the following diagnoses is most likely? [B2 Q14]
a. Crohn’s colitis
b. Ulcerative colitis
c. Ischaemic colitis
d. Infectious colitis
e. Pseudomembranous colitis
Ischaemic colitis
Crohn’s colitis is relatively unlikely due to lack of prior history or small bowel involvement
and age of the patient. Ulcerative colitis and pseudomembranous colitis are both unlikely as
the rectum is usually involved in these two conditions. Infectious colitis does not normally
affect the left-sided colon only, regardless of the underlying pathogen. Ischaemic colitis is the
most likely diagnosis of those listed. It typically affects a segment of bowel, with most cases
having left-sided colonic involvement.
A 64-year-old man presents to A&E with onset of severe watery diarrhoea and abdominal pain.
An AXR is performed which shows dilated large bowel and nodular haustral fold thickening.
The patient has a CT scan with oral and intravenous contrast. The CT scan shows large bowel
dilatation with diffuse bowel wall thickening. Some of the oral contrast given has become
trapped between the oedematous haustral folds, causing alternating bands of high and low
attenuation. What is the most likely underlying diagnosis? [B1 Q55]
A. Ulcerative colitis.
B. Crohn’s colitis.
C. Ischaemic colitis.
D. Pseudomembranous colitis.
E. Bacillary dysentery.
Pseudomembranous colitis
The CT sign of oral contrast becoming trapped between thickened oedematous haustral folds
is the ‘accordion’ sign, which is highly suggestive of pseudomembranous colitis. Bowel wall
thickening, peri-colonic stranding of fat, and ascites may be seen in all forms of colitis.
Pseudomembranous colitis can be segmental but is more commonly a pancolitis.
A 76-year-old male on ITU has a CT abdomen and pelvis for the investigation of abdominal
pain, pyrexia, and diarrhoea. The CT reveals 12mm diffuse large bowel wall thickening with
intense mucosal enhancement and low attenuation of the submucosa involving the entire colon
including the rectum, and a small volume of ascites. Which one of the following diagnoses is
the most likely to explain the above findings? [B2 Q47]
a. Crohn’s colitis
b. Pseudomembranous colitis
c. Ischaemic colitis
d. Yersinia
e. Giardiasis
Pseudomembranous colitis
Pseudomembranous colitis results from overgrowth of Clostridium difficile most commonly
due to broad spectrum antibiotic use in the hospital population. Ascites is often present in
severe cases and wall thickening >10 mm is highly suggestive of this diagnosis. A layered
pattern of enhancement is often present in severe cases with oedema in the submucosa
producing low attenuation in the wall, deep to the enhancing mucosa. The accordion sign is
caused by marked submucosal oedema producing thickening of the colonic haustra. The rectum
is involved in most cases but any location within the large bowel may be involved.
A 60-year-old woman presents with abdominal pain and diarrhoea six weeks post-hip
replacement surgery, with her recovery being complicated by a hospital-acquired pneumonia.
CT is performed. When considering a diagnosis of pseudomembranous colitis (PMC) which is
the most common finding? [B3 Q5]
A. Intense mucosal enhancement
B. Enlarged peri-colic blood vessels
C. An irregular or discontinuous mucosal lining
D. Peri-colonic fat stranding
E. Colonic wall thickening
Although all the above are recognised findings in PMC, colonic wall thickening from minor to
gross thickening is the most common feature. The relatively minimal pericolonic stranding with
marked colonic wall thickening may help to distinguish PMC from other colonic pathologies.
A 70-year-old hospitalized male patient presents with watery diarrhoea and abdominal pain.
CT of the abdomen demonstrates marked circumferential bowel wall thickening involving the
entire colon, with minimal peri-colonic stranding and a small amount of ascites. The small
bowel appears normal. What is the most likely diagnosis? [B4 Q2]
a. Crohn’s disease
b. ischaemic colitis
c. diverticulitis
d. pseudomembranous colitis
e. ulcerative colitis
Pseudomembranous colitis
Pseudomembranous colitis is an acute infectious colitis caused by Clostridium difficile and its
toxins A and B; this pathogen has become increasingly common largely due to widespread use
of broad-spectrum antibiotics. The commonest CT finding is of colonic wall thickening (due
to mural oedema and the presence of pseudomembranes), which is typically greater than in
other causes of colitis apart from Crohn’s disease. Pericolonic inflammatory changes are
disproportionately mild relative to the marked wall thickening. Ascites is common, and this,
together with the lack of small bowel involvement, can help to distinguish pseudomembranous
colitis from Crohn’s colitis. Ischaemic colitis demonstrates less wall thickening, and is usually
segmental, tending to affect the watershed areas of the colon.
A 35-year-old male with known ulcerative colitis presents to A&E with severe abdominal pain,
pyrexia, and diarrhoea. There is no peritonism. Toxic megacolon is suspected clinically. Which
one of the following is the most appropriate as first line imaging? [B2 Q30]
a. CT
b. Plain abdominal film
c. Double contrast barium enema
d. Single contrast water-soluble enema
e. Targeted bowel ultrasound
Plain abdominal film
Toxic megacolon is a complication of ulcerative colitis, Crohn’s and other forms of acute
colitis. It has a poor prognosis with up to 20% mortality. Plain abdominal radiography should
be the first line investigation for suspected toxic megacolon and can be repeated 24 or 48 hourly
if necessary. It can often confirm the diagnosis without the need for CT, which is especially
useful when considering radiation dose issues in this group of young patients. Typical features
on plain film include transverse colon dilatation >5.5 cm, loss of normal haustral folds,
thumbprinting of the colon and the presence of mucosal islands (pseudopolyps). CT better
demonstrates potential complications of toxic megacolon such as perforation of the bowel.
As part of an investigation for altered bowel habit, a 32-year-old female has a double contrast
barium enema performed. Findings include deep and superficial aphthous ulceration from the
caecum proximally to the sigmoid colon and the presence of pseudodiverticula. Which one of
the following is most likely? [B2 Q31]
a. Crohn’s disease
b. Ulcerative colitis
c. Tuberculosis
d. Yersinia
e. Lymphoma
Crohn’s disease
These features are highly suggestive of Crohn’s disease. Signs on double contrast barium
enema that favour a diagnosis of Crohn’s disease over ulcerative colitis include apthoid ulcers,
deep ulcers, discontinuous ulceration, rectal sparing, pseudodiverticulae, fistulae and abscess
formation. Ulcerative colitis can be suggested by rectal involvement, continuous pathology
with no skip lesions and the presence of mucosal granularity. However, these features may also
be present in Crohn’s and are not specific for ulcerative colitis. Although tuberculosis is a
mimic, colonic involvement in this pattern is uncommon compared with Crohn’s disease.
A 34-year-old woman presents with bloody diarrhoea and abdominal pain. Which feature on
barium enema favours a diagnosis of ulcerative colitis rather than Crohn’s disease? [B4 Q29]
a. thickened ileocaecal valve
b. circumferential wall involvement
c. fistula formation d. skip lesions
e. normal rectum
Circumferential wall involvement
Ulcerative colitis and Crohn’s disease are idiopathic inflammatory diseases of the bowel.
Ulcerative colitis predominantly involves the mucosa and submucosa, and characteristically
produces continuous, circumferential involvement of the colon. Crohn’s disease produces
transmural inflammation, may affect the entire gastrointestinal tract, and is characterized by
eccentric and discontinuous involvement. Typical features of ulcerative colitis include
predominantly left-sided colonic involvement with rectosigmoid involvement in 95% of cases,
a patulous ileo-caecal valve and shallow ulceration. Typical features of Crohn’s disease include
skip lesions (discontinuous disease), terminal ileal involvement with a thickened ileocaecal
valve and fistula formation.
A seven-year-old boy on chemotherapy for acute leukaemia develops severe right iliac fossa
pain and diarrhoea. CT shows ascending colon and caecal wall thickening, with inflammation
extending to involve the appendix and terminal ileum and fat stranding in the adjacent
mesentery. The most likely diagnosis is: [B2 Q6]
a. Typhlitis
b. Crohn’s disease
c. Acute appendicitis
d. Necrotising enterocolitis
e. Acute leukaemic infiltration
Typhlitis
Typhlitis, or neutropaenic enterocolitis, is acute inflammation of the caecum, ascending colon,
terminal ileum, or appendix. It is typically described in children with neutropaenia secondary
to lymphoma, leukaemia, and immunosuppression. Concentric, often marked, bowel wall
thickening with pericolic inflammatory changes is typical, and such changes in a young,
immunosuppressed child should raise suspicion of typhlitis as a cause. Perforation is a risk
factor and therefore contrast examinations are usually avoided.
A 45-year-old woman with a previous history of treatment for advanced carcinoma of the
cervix 8 years earlier presents with constipation and rectal bleeding. She undergoes CT of the
abdomen and pelvis, which demonstrates a narrowed rectum with symmetrical wall thickening,
perirectal inflammatory changes, thickening of the perirectal fascia and an increase in the AP
diameter of the presacral space. What is the most likely diagnosis?
a. colorectal carcinoma
b. ulcerative colitis
c. radiation injury of the rectum
d. Hirschsprung’s disease
e. lymphoma
Radiation injury of the rectum
Gastrointestinal complications following external radiotherapy are becoming more frequent as
survival rates of patients with abdominal cancer improve, and they may present up to 15 years
following irradiation. The colon and rectum are commonly affected following irradiation for
pelvic and genitourinary tract malignancies. Chronic radiation colitis usually presents with
strictures, whereas rectal injury manifests as a narrowed, thickened, poorly distensible rectum,
with proliferation of the perirectal fat and thickening of the perirectal fascia. There is an
increased incidence of colorectal carcinoma following pelvic irradiation, but a short irregular
segment of narrowing would be more likely. Inflammatory bowel disease may result in
circumferential rectal wall thickening with widening of the presacral space but is less likely
given this history. Hirschsprung’s disease usually presents in early childhood. The rectum is
an uncommon site for gastrointestinal lymphoma
Familial Adenomatous Polyposis
A 20-year-old male with a recent history of medulloblastoma now presents with vague
abdominal pain, PR bleeding, and weight loss. Innumerable colonic polyps are demonstrated
on colonoscopy. What is the most likely unifying diagnosis? [B1 Q74]
A. Familial adenomatous polyposis.
B. Turcot syndrome.
C. Gardner syndrome.
D. Lynch syndrome.
E. Chronic inflammatory bowel disease
Turcot syndrome
FAP is a rare autosomal dominant condition resulting in the growth of hundreds of
adenomatous polyps in the large bowel. Clinical symptoms commence from the third decade
and include abdominal pain and PR bleeding. Colorectal cancer develops in almost all before
the age of 40 years. Turcot syndrome is characterized by the association of colonic polyps like
FAP and central nervous system tumours, typically medulloblastoma and glioblastoma
multiforme. The combination of intestinal polyposis (identical to FAP) and numerous osteomas
and epidermal cysts is typical of Gardner syndrome. Lynch syndrome is hereditary non-
polyposis colorectal cancer. Colorectal carcinoma occurs earlier than in the average population.
There is also an association with ovarian and endometrial malignancy. Chronic inflammatory
bowel disease is not associated with CNS malignancy.
A 25-year-old male presents with abdominal cramps and pain with rectal bleeding.
Colonoscopy is normal. CT enteroclysis is performed as part of the investigation, which reveals
multiple sessile polyps throughout the jejunum and ileum. Subsequent biopsies reveal these
polyps to be hamartomas. Which one of the following syndromes is he most likely to be
diagnosed with? [B2 Q19]
a. Peutz–Jeghers
b. Cowden’s
c. Turcot’s
d. Familial polyposis
e. Gardner’s
Peutz–Jeghers
Peutz–Jeghers syndrome is most consistent with these findings. It is an autosomal dominant
syndrome but often arises as a spontaneous mutation. Hamartomas are found throughout the
gastro-intestinal tract, except for the oesophagus. The polyps have almost no malignant
potential, but life expectancy is decreased due to associated cancers arising in the stomach,
duodenum, colon, and ovary. Gardner’s syndrome and familial polyposis are both associated
with small bowel adenomas in approximately 5% of cases. Cowden’s syndrome does involve
hamartomatous polyps, but these are typically rectosigmoid, and small bowel involvement is
not a feature. Small bowel polyps are not a feature of Turcot’s syndrome.
A 25-year-old woman presents with cramping abdominal pain and bleeding per rectum. On
examination she has mucocutaneous pigmentation of her mucous membranes and face. A
small-bowel follow-through examination demonstrates small-bowel intussusception. Which
other finding is most likely to be demonstrated? [B4 Q20]
a. separation and displacement of small bowel loops
b. localized outpouching of the antimesenteric border of the distal ileum
c. generalized irregular fold thickening
d. multiple filling defects in the small bowel
e. generalized dilatation of the small bowel
Multiple filling defects in the small bowel
Peutz–Jegher syndrome is characterized by multiple benign hamartomatous intestinal polyps
and mucocutaneous pigmentation. It is familial in 50% of cases, with an autosomal dominant
inheritance, and sporadic in 50%. It is the most common polyposis syndrome to involve the
small intestine, and frequently presents with intussusception. Typical findings are of multiple
hamartomatous polyps in the small bowel, and less commonly the colon and stomach. Patients
are at increased risk of gastrointestinal malignancy, but also of tumours of the pancreas, breast,
ovary, endometrium and testis.
A 17-year-old female undergoes screening colonoscopy and is found to have multiple
adenomatous polyps throughout the colon. OGD and biopsy reveal multiple hamartomas of the
stomach and duodenum. She subsequently has investigation for a painful jaw that reveals a
1cm round, discrete, dense lesion in the mandible. Which one of the following syndromes is
the most likely underlying diagnosis? [B2 Q20]
a. Lynch syndrome
b. Cronkhite–Canada syndrome
c. Familial adenomatous polyposis
d. Gardner’s syndrome
e. Peutz–Jegher syndrome
**Gardner’s syndrome **
Gardner’s syndrome is an autosomal dominant condition with colonic polyps present in all
patients. Small bowel, duodenal and stomach polyps are also a feature. Extra-intestinal features
include osteomas of membranous bone (typically the mandible as described in the question),
other soft-tissue tumours and periampullary carcinomas. Osteomas are not a feature of the other
conditions. Cronkhite–Canada syndrome and Peutz–Jegher syndrome are associated with
multiple hamartomatous polyps of the colon and stomach. Cronkhite-Canada syndrome is a
sporadic non-familial disorder. Lynch syndrome, or hereditary nonpolyposis colorectal
carcinoma (HNPCC), is associated with increased risk of colorectal adenomas and other
malignancies such as endometrial and other gastro-intestinal tract malignancies.
A 26-year-old female presents with a 1-day history of right iliac fossa (RIF) pain. She is mid-
cycle and prone to mittleschmerz-type pain but reports that this pain is more severe than
previously. Serum inflammatory markers are elevated. Clinical examination reveals tenderness
in the RIF, but no rebound. Due to the compounding gynaecological history, a CT is requested.
This reveals a thickened caecum and thickened appendix, which appears to have a defect in the
wall on the multiplanar reformatted images. There is a calcified density present in the orifice
of the appendix. There is a loculated fluid collection adjacent to the appendix, which has air
bubbles within it. There is also fluid in the pelvis. A perforated appendix is removed at surgery.
Which of the CT findings is most specific for detecting a perforated appendix? [B1 Q9]
A. Presence of a faecolith.
B. Identification of a wall defect.
C. Fluid in the pelvis.
D. Adjacent abscess formation.
E. Enlarged regional lymph nodes.
Adjacent abscess formation.
Abscess formation has been found to be the most specific finding in appendiceal perforation,
along with extraluminal gas and small bowel ileus. Abscess formation is also one of the least
sensitive findings. Regional mesenteric lymph nodes are the most sensitive but are reasonably
non-specific. A focal wall defect, if seen, is reasonably sensitive and specific. Appendicolith is
only found in 50% on CT and has a specificity of 70%.
An ultrasound of the abdomen is performed on a 21-year-old female presenting to A&E with
acute right iliac fossa pain, pyrexia, tenderness and guarding. Which one of the following
findings would suggest perforation of the appendix? [B2 Q56]
a. Appendix diameter of 8mm
b. Appendix wall thickness of 4mm
c. Decreased resistance of arterial waveform
d. Loss of visualisation of hyperechoic submucosa
e. Increased echogenicity of surrounding fat
Loss of visualisation of hyperechoic submucosa
The use of ultrasound for the diagnosis of acute appendicitis is particularly useful in children
and women of child-bearing age. Findings indicating acute appendicitis include a tubular non-
compressible blind-ending structure with diameter >6 mm and wall thickness >2 mm, although
these signs do not necessarily indicate perforation. Features suggesting perforation include a
fluid collection adjacent to the appendix, gas bubbles near the appendix and loss of
visualisation of the submucosal layer.
A 23-year-old man presents with acute lower abdominal pain. An abdominal radiograph
demonstrates a rounded, laminated calcific density projected over the right lower quadrant.
What is the approximate likelihood of a diagnosis of acute appendicitis? [B4 Q31]
a. 10%
b. 30%
c. 50%
d. 70%
e. 90%
90%
A laminated calcified appendicolith is seen in only 7–15% of patients with acute appendicitis.
However, the presence of acute abdominal pain with an appendicolith on abdominal plain film
indicates a 90% probability of acute appendicitis, and also indicates a high probability of
gangrene/ perforation. Other plain film signs of acute appendicitis include caecal wall
thickening, small bowel obstruction and focal extraluminal gas collections
A 65-year-old man presents with a several-week history of lower abdominal pain and diarrhoea.
On examination he has tenderness and guarding in the left lower quadrant. On contrast-
enhanced CT, the inferior mesenteric vein is dilated, with a thin rim of enhancement around a
central area of low density. What is the most likely additional pathology demonstrated on the
CT? [B4 Q13]
a. sigmoid diverticulitis
b. appendicitis
c. Crohn’s disease
d. pancreatitis
e. caecal malignancy
Sigmoid diverticulitis
The inferior mesenteric vein provides venous drainage for the rectum, sigmoid and descending
colon, and is a potential route of spread of neoplastic and inflammatory conditions. Inferior
mesenteric vein thrombosis may occur secondary to an inflammatory process, most commonly
diverticulitis, or malignancy. Other potential causes include hypercoagulable states, surgery,
trauma and bowel obstruction. Appearances are of an enlarged vein with rim enhancement
surrounding central low-density thrombus. Superior mesenteric vein thrombosis is much more
common (95% of mesenteric venous thrombosis) and may follow an inflammatory or
neoplastic process affecting the small intestine, caecum, and ascending and transverse colon.
A 40-year-old man is admitted to the surgical ward with acute abdominal pain and subsequently
a CT abdomen and pelvis is requested. The findings include a 3cm oval mass with central fat
density adjacent to the sigmoid colon and with associated fat stranding. Which one of the
following is the most likely diagnosis? [B2 Q5]
a. Diverticulitis
b. Epiploic appendagitis
c. Mesenteric lymphadenitis
d. Meckel’s diverticulitis
e. Infected enteric duplication cyst
Epiploic appendagitis
Epiploic appendagitis is inflammation of one of the epiploic appendages of the colon, with the
sigmoid being the commonest site. It typically presents with acute abdominal pain and is an
important radiological diagnosis as it can often mimic appendicitis, and management is
conservative. The diagnosis is usually made on CT with the features described in the question.
Ultrasound is rarely used for diagnosis, and features include a non-compressible hyperechoic
mass with hypoechoic margins.
A usual fit and well 30-year-old female presents with left-sided abdominal pain. Normal uterus
and ovaries are identified. In the left iliac fossa, there is a 4-cm hyperechoic mass which is non-compressible between the colon and abdominal wall. No colonic wall changes are identified.
There is no free fluid. The lesion is surrounded by a hyperechoic border with no Doppler flow.
Which is the most likely diagnosis? [B3 Q14]
A. Epiploic appendigitis
B. Diverticulitis
C. Omental infarction
D. Omental metastases
E. Mesenteric panniculitis
Self-limiting condition with fat density mass 1-4cm between colon and abdominal wall. The
absence of blood flow on Doppler due to torsion helps distinguish from acute diverticulitis.
Omental infarcts are usually large, right-sided and cake-like in appearance.
A 34-year-old man presents with acute left lower quadrant pain following unaccustomed
exercise. CT of the abdomen demonstrates a 2.5 cm oval lesion with attenuation value of–60
HU abutting the sigmoid colon, with surrounding inflammatory changes. The sigmoid colon
itself appears normal. What is the most likely diagnosis? [B4 Q9]
a. omental infarction
b. diverticulitis
c. epiploic appendagitis
d. liposarcoma
e. appendicitis
Epiploic appendagitis
Acute epiploic appendagitis is thought to result from torsion of one of the fatty epiploic
appendages arising from the serosal surface of the colon. It usually occurs in young men,
presenting as acute lower quadrant pain, and is associated with obesity and unaccustomed
exercise. Typical CT findings are of an oval peri-colonic fat density lesion of ,5cm, with
surrounding inflammatory changes, most commonly in the sigmoid, descending or right hemi-
colon. Right-sided epiploic appendagitis may be mistaken clinically for appendicitis. Omental
infarction typically appears as a larger, heterogeneous lesion, usually affecting the caecum or
ascending colon. Acute diverticulitis usually occurs in older patients and is associated with
colonic diverticula and wall thickening. Liposarcoma is rare but is included in the differential
of a fat-containing intra-abdominal mass.
A 39-year-old male complains of severe, colicky left lower abdominal pain and rectal bleeding.
He has experienced intermittent abdominal pain for the last 3–4 months. There is no previous
history of medical problems. On examination he has left lower abdominal tenderness without
signs of peritonism. A CT examination is performed which reveals a focal intraluminal
abnormality, with the appearance of a mass within the sigmoid colon. There are concentric
rings of soft tissue and fatty attenuation giving a ‘target’ like appearance. Mesenteric vessels
are seen to course into the lesion. At the most distal point of the abnormality there is a more
discrete low attenuation mass measuring approximately 3 cm in size of fatty attenuation. The
large bowel distal to the sigmoid lesion is collapsed and proximal to the lesion there are
multiple loops of small bowel and a dilated colon. What is the most likely underlying pathology
for this condition? [B1 Q61]
A. Benign tumour.
B. Malignant tumour.
C. Inverted diverticulum.
D. Idiopathic.
E. Inflammatory bowel disease.
Benign tumour
The patient has presented with a colo-colic intussusception secondary to a lipoma of the colonic
wall. The intussusception has been complicated by large bowel obstruction.
Intussusception is caused by prolapse of a portion of the bowel into the lumen of the adjoining
bowel lumen segment. Intussusception is most commonly a disease of young children and most
commonly occurs in an ileocolic location. In children, over 90% of intussusceptions are
idiopathic and no lead point is identified. In adult patients approximately 80% of
intussusceptions are caused by lead point lesions.
The CT appearances described are typical of an intussusception. The low attenuation distal
lead point represents the lipoma. Ultrasound is often used to diagnose intussusception in
children and will show a mass with echogenic rings representing the fat in the invaginated
mesentery.